Series 66

अब Quizwiz के साथ अपने होमवर्क और परीक्षाओं को एस करें!

Which of the following vehicles make use of the unified estate tax credit? 1Bypass trust 2Generation-skipping trust 3Living trust 4Simple trust A) III and IV B) I and II C) II and III D) I and IV

Both the bypass trust and the generation-skipping trust are tools used by estate planners to reduce estate taxes. They do so by passing the amount in the unified credit (currently $5.34 million for 2014) to heirs other than the spouse, usually grandchildren in the case of the GST.

If the current risk-free rate is 4%, and the expected return from the market is 10%, what return should we expect from a security that has a beta of .9? A) 9% B) 9.4% C) 6.4% D) 13%

Required return = 4% + ([10% - 4%] × .9) = 4% + (6% x .9) = 4% + 5.4% = 9.4%.

The sale of a life insurance policy in the secondary market by a terminally ill individual is known as A) an unethical trade practice B) a trade in the OTC market C) a viatical settlement D) a vertical settlement

A viatical settlement is the sale of a life insurance policy to a third party (the investor). The owner (viator) of the life insurance policy sells the policy for an immediate cash benefit. The buyer becomes the new owner of the life insurance policy, pays future premiums, and collects the death benefit when the insured dies. At one time, most viatical settlements were from people with a life-threatening illness. Now, individuals who are not facing a health crisis may sell their life insurance policies to get cash

A customer needs $10,000 to pay for a new house within the next year. His agent suggests that he invest in a stock that has been performing extremely well the past year and assures the customer that he cannot go wrong. According to the Uniform Securities Act, this is an unethical business practice an example of guaranteeing a profit an example of flamboyant language an unsuitable investment A) I and III B) II and IV C) I, II and IV D) I, II, III, and IV

All the choices listed are true. There is nothing in the LEM about flamboyant language, but there should be enough examples of what is fair and ethical for you to realize that flamboyant or exaggerated statements would be against the spirit of the law if not actually a violation. Use that kind of common sense on the real exam.

Programs allowing for the direct pass-through of losses and income to investors include all of the following EXCEPT A) S corporations B) oil and gas drilling direct participation programs C) new construction real estate direct participation programs D) REITs

REITs allow for the direct pass-through of income but not losses. The other choices are forms of business which allow for pass-through of income and losses.

If Janet established a Coverdell Education Savings Account for her grandson, in each successive year, she may contribute A) $3,000.00 B) $1,000.00 C) $4,000.00 D) $2,000.00

Under current regulations, the maximum contribution to a Coverdell Education Savings Account is $2,000 annually.

The term used to describe a customer initiated order that includes all of the details except time and price is A) discretionary B) solicited C) limit D) unsolicited

When the order to buy or sell is initiated by the customer, it is considered to be an unsolicited order. Discretionary orders leave the decision as to what security, how much, and buy or sell up to the designated agent. Limit orders carry a specified price and time.

Which of the following items would be found on a family balance sheet? A) Income taxes paid B) Dividends and interest received C) Annual salary D) Spouse's engagement ring

A balance sheet, whether for a family or a business, shows assets and liabilities, not income and expenses. The ring is certainly an asset; the others are income or expenses

Under the terms of the Uniform Securities Act, which of the following is an investment adviser for purposes of state regulatory jurisdiction? A) A federal covered adviser with clients in the state B) A commercial bank with a place of business in the state that advises clients on banking matters C) An investment advisory subsidiary of a bank holding company located in the state that manages $20 million in assets D) An accountant located in the state who offers general securities advice as an incidental part of his business

A bank holding company's investment advisory subsidiary that manages $20 million in assets is an investment adviser subject to the Uniform Securities Act (USA). Under the language of the USA, a commercial bank is excluded from the definition of investment adviser whereas a bank holding company subsidiary is not. While a federal covered adviser is an investment adviser in practice (that is, it performs the functions of an adviser), it is excluded from the definition of an investment adviser under the USA to avoid duplicate regulation. An accountant located in the state that offers general securities advice as an incidental part of his business is not an investment adviser.

A customer's portfolio has a beta coefficient of 1.1. If the overall market increases by 10%, the portfolio's value is likely to A) decrease by 10% B) decrease by 11% C) increase by 10% D) increase by 11%

A beta of 1.1 means the portfolio is considered to be 1.1 times more volatile than the overall market. If the market is up 10%, the portfolio with a beta of 1.1 is likely to be up 11%.

Which of the following may be required by the Administrator to post surety bonds? An agent who has discretion over client funds and securities A broker-dealer who has custody of, or discretion over, client funds and securities An investment adviser who has custody of, or discretion over, client funds and securities A) I only B) III only C) I, II, and III D) I and III

A broker-dealer, investment adviser, or agent who has discretion over or, in the case of broker-dealers and advisers, custody of funds or securities may be required to post a bond. U3LO5

An investor wishing to buy US Treasury bonds receives a quote from the dealer of 98.16. This represents A) an indication of falling interest rates B) the bid price C) a discount D) the offer price

A dealer's quotes consist of the bid and the offer (ask). The bid price is what the dealer will pay a customer to purchase a security, and the offer is the dealer's selling price. In this case, the client wishes to purchase bonds, so the 98.16 represents the price the dealer is asking for them. Yes, the quote is a discount, but the better answer for this question is offer

If a client prefers mutual fund investments in companies that primarily generate capital appreciation to companies that pay a steady dividend, what type of mutual fund and associated investment objective would you recommend? A) An income fund B) A growth fund C) A growth and income fund D) An index fund

A growth mutual fund invests in stocks that are growing rapidly and stresses capital appreciation rather than income. The key is that the growth and appreciation are synonymous.

The Securities Exchange Act of 1934 defines a market maker is A) a person who buys and sells securities for her own account or for the accounts of others B) a dealer who, with respect to a security, holds himself out as being willing to buy and sell that security for his own account on a regular or continuous basis C) an agent for the issuer D) an agent whose clients are institutions

A market maker is a dealer who holds himself out as being willing to buy or sell a security at a quoted price on a regular and continuous basis.

A married couple in their early 50s saving for retirement would most likely have which of the following objectives? A) High risk, moderate safety, low liquidity B) Low risk, high safety, high liquidity C) Moderate risk, moderate safety, low liquidity D) Moderate risk, low safety, high liquidity

A middle-aged couple planning for their retirement is most likely interested in moderate risk, moderate safety, and low liquidity. A couple of their age should be planning for retirement and the demands for liquidity should be low; they need to take moderate risk to earn above-inflation returns. Moderate safety is appropriate for a middle-aged couple. Additionally, a middle-aged couple should not be invested in high-risk securities.

A mutual fund's expense ratio is found by dividing its expenses by its A) dividends B) income C) public offering price D) average annual net assets

A mutual fund's expense ratio is calculated by dividing its expenses by its average annual net assets. For example, if the fund had net assets of $100 million and its annual expenses are $1 million, the expense ratio is $1 million divided by $100 million = 1%.

A mutual fund's computed NAV on April 24 is $100 per share. On April 25, the portfolio realized gains of $2 per share, and enjoyed $1 per share in unrealized appreciation. What would the NAV be on April 26 assuming an unchanged market? A) $100 per share B) $103 per share C) $102 per share D) $101 per share

A mutual fund's net asset value per share (NAV) is the fund's total assets minus total liabilities (net asset) divided by the number of shares outstanding. The major asset is the fund's portfolio. Portfolio securities are carried at their value as of the close of the markets (4PM ET). As a result, unrealized appreciation (and depreciation) are part of the NAV. Therefore, when that gain (or loss) is realized, paper profit (or loss) is now real and there is no change to total assets. In the subject question, the realization of the $2 per share gain has no effect, but, the new $1 in unrealized appreciation increases the NAV by that amount.

A client with a net worth of $5 million is compensating an investment adviser with a performance-based fee. According to the Investment Advisers Act of 1940, this arrangement must be based on A) the S&P 500 index performance B) capital gains minus capital losses, including both realized and unrealized gains and losses C) This arrangement is not permitted because the client has not met the minimum invested assets requirements D) a period of no less than 6 months

A performance-based fee must be based on capital gains minus capital losses, include both realized and unrealized gains and losses and must reflect a time period of no less than 12 months. This client is well above the minimum net worth requirements of more than $2.1 million. The rule requires that the performance be measured against a recognized benchmark but does not specify one.

An agent has a client who calls in with the following instructions: "Please place a sell stop order at $33 for my 100 shares of ABC common stock." A week later, the agent receives inside information that ABC's upcoming earnings report will be disastrous. Three days after that, the report is released and the stock plunges, falling to $33. Can the client's stop order be executed? A) Yes, because the order was received prior to the agent receiving the information B) Yes, but only if the execution price is higher than $33 C) No, because the order isn't triggered until the price falls below $33 D) No, because the agent was in possession of material inside information

A sell stop order at $33 becomes a market order once the stock's price declines to $33 (or less). At that point, the order may be executed at, above, or below the stop price. The agent came into possession of the inside information after the order was received, so there is no concern that the client was benefiting from the agent's knowledge.

A customer opens a margin account with a broker-dealer and signs a loan consent agreement. The loan consent agreement allows the firm to A) lend the customer money B) hypothecate securities in the account C) loan out the customer's margin securities D) commingle the customer's securities with securities owned by the firm

A signed loan consent agreement permits a firm to loan out a customer's margin securities. This is the only part of the margin documentation that is optional.

If an investor wants to invest in the electronics industry but does not want to limit his investments to only one or two companies, which type of fund would be most suitable? A) Bond B) Specialized C) Money market D) Hedge

A specialized or sector fund invests 25% or more of its assets in a particular region or industry.

An investor would have to pay the alternative minimum tax when A) it exceeds the investor's regular income tax B) the investor has received income from a limited partnership C) there are tax-preference items reported on the tax return D) the investor's capital gains exceed 10% of total income

A taxpayer must pay the alternative minimum tax in any year that it exceeds regular tax liability. Tax-preference items are re-input in figuring AMT, but the AMT is paid only if that amount is higher than the regular income tax.

The SEC has determined that advertising regarding past recommendations made by investment advisers is misleading if 1results do not reflect the deduction of fees 2actual market conditions during the referenced period are not disclosed 3the advertisement did not reflect performance for a minimum period of 3 years 4the advertisement did not disclose that it applied to only a specific group of clients A) I, II, and IV B) II and IV C) I and II D) I, II, III, and IV

Advertising that reflects past performance must show a minimum period of 1 year, not 3. All investment advisers' advertising must reflect deduction of fees; disclose the specific group of clients to which it applies, if applicable; and state actual market conditions during the referenced period.

Under the Investment Advisers Act of 1940, an adviser who has custody of a client's funds must 1notify a client when the client's funds are moved to another location 2segregate client's funds and keep them identified by client 3not move the client's funds without prior notification and specific written authority from the client A) II and III B) I and III C) I, II, and III D) I and II

Advisers who have custody must segregate a client's securities and keep them in a safe place, deposit client funds in bank accounts that contain only client funds (may be combined in one account, but complete records must be kept), report to clients at least every 3 months with a statement, and annually arrange for an unannounced audit by an independent accountant that will report the audit results to the SEC. All clients must be notified in writing of the location of their securities or funds and of any changes to the location. It is not necessary to notify the client before the move to obtain the client's specific written authority to move the fund. The original custodial agreement includes that authority at the discretion of the adviser. U7LO2

A complex trust has the following income for the year: $1,500 in taxable interest, $2,000 in dividends (reinvested in the stock), and $3,000 in tax-exempt interest. In addition, the portfolio realized $3,500 in capital gains that were reinvested in the corpus. What is the distributable net income (DNI) for the trust? A) $6,500 B) $4,500 C) $10,000 D) $1,500

All investment income, regardless of source, will be considered DNI and will be included in the taxable income calculation to the trust unless distributed. That portion of the DNI representing tax-exempt interest maintains its tax-free status. Reinvested capital gains are not part of a trust's DNI. The computation is: $1,500 in taxable interest + $2,000 in dividends (reinvestment means nothing here) + $3,000 in tax-exempt interest. This is a total of $6,500 of DNI. When distributed, only $3,500 will be taxable.

Under the Investment Advisers Act of 1940, a registered investment adviser may A) imply SEC approval or sponsorship because of registration B) use the statement "registered with the SEC" in advertisements C) use the initials RIA after its name on a business card D) imply SEC approval or sponsorship because of passing an exam.

Although an investment adviser registered with the SEC may state that fact, a registered investment adviser may not use the title in any way to suggest or imply that the SEC sponsors or approves the adviser. The title in no way indicates that the adviser's abilities or qualifications have been approved. Because RIA is not an academic designation, it may not be used as such.

A state-registered investment adviser suddenly incurs a liability that materially affects its net worth, causing it to drop below the required minimum. Which of the following statements is TRUE? A) The ​investment adviser is not required to file​ ​an amendment to its registration with the Administrator. B) The​ ​investment adviser must notify the Administrator​ by the close of business on the following business day​. C) The investment adviser must notify the Administrator promptly. D) The ​investment adviser ​must increase its surety bond to make up the deficiency.

Although most notifications involving emergency type situations require prompt notification, when an investment adviser's net worth is below the requirement, the NASAA Model Rule is a bit different. Unless otherwise exempted, as a condition of the right to transact business in the state, every investment adviser registered with the state shall, by the close of business on the next business day, notify the Administrator if such investment adviser's net worth is less than the minimum required. After transmitting such notice, each investment adviser shall file by the close of business on the next business day after that, a report with the Administrator of its financial condition.

One of your clients has called you to discuss an interesting investment opportunity discovered on one of the LinkedIn groups she participates in. Which of the following factors might increase the likelihood that this is a scam? 1A registration statement with the SEC is available on the website of the proposed investment 2The purchase money must be wired to an offshore account 3One of the members of the group is a principal in the company being offered 4Bonus shares are offered for recruiting friends into the deal A) II, III, and IV B) I, II, III, and IV C) I and III D) II and IV

Although not foolproof, the existence of an available SEC registration statement greatly reduces the likelihood that a deal like this is a scam. The other choices are certain red flags.

One of the features of an index annuity is the ability for the principal value to increase based on the performance of the specified index. Which of the following is NOT used as a method to compute the amount of interest to be credited to the account? A) Annual reset B) Participation rate C) High-water mark D) Point to point

Although the participation rate is a component of the computation, it is not a method of computing the interest credit. In the annual reset index method, interest, if any, is determined each year by comparing the index value at the end of the contract year with the index value at the start of the contract year. Interest is added to the annuity each year during the term. Using the high-water mark, the index-linked interest, if any, is decided by looking at the index value at various points during the term, usually the annual anniversaries of the date the annuity was purchased. The interest is based on the difference between the highest index value and the index value at the start of the term. Interest is added to the annuity at the end of the term. And finally, with the point-to-point method, the index-linked interest, if any, is based on the difference between the index value at the end of the term and the index value at the start of the term. Interest is added to the annuity at the end of the term. In each of these, the insurance company will specify the participation rate (what percentage of the increase will be credited) and a cap rate (the maximum amount to be credited).

Under both state and federal law, there are a number of exclusions from the definition of investment adviser. Which of the following would not qualify for an exclusion? A) A publisher of a newsletter that is paid to make reports to be used in the sale of specific securities B) A personal injury attorney who recommends that clients consult with a CFP® for advice on how to deal with the large settlements they receive C) A CPA who gives high tax bracket clients a chart showing the tax-equivalent yield of municipal bonds D) An economist who teaches a course in fundamental analysis at a local community college

Although there is an exclusion for publishers, it must be of general and regular circulation and not be the recipient of compensation from the issuers of any securities covered

An owner of an equity index annuity would be wise to use the high-water crediting method if the underlying index was expected to A) be volatile. B) decline. C) change its objective. D) remain steady.

An advantage of the high-water crediting method is that the interest is calculated using the highest value of the index during the term. Therefore, in a volatile market, where prices are going up and down, it picks up the highest price. DUPLICATE

An agent is assisting a prospective client in opening an account. The individual refuses to provide his net worth and annual income. The agent should A) in the absence of company policy to the contrary, open the account but limit transactions to unsolicited orders B) seek permission to consult with the client's fiduciary team, including accountants and attorneys to obtain the financial information C) proceed with opening the account, but limit recommendations to conservative investments D) refuse to open the account

An agent must attempt to obtain client financial information. The broker-dealer, through its principals, may decide whether to accept business from a client refusing to provide financial information. In the absence of financial information, neither the firm nor the agent has the means to determine client suitability. Thus, the firm may only accept unsolicited orders from this client.

Tim, Jim, and Kim are equal partners in TJK Investment Advisers, a general partnership. Tim decides to sever his relationship with the other partners and work for a different firm. When, if at all, must the clients of TJK be notified of Tim's departure? A) TJK must notify its clients of Tim's departure within a reasonable period after his severance from the firm. B) TJK must notify its clients of Tim's departure within 15 days of Tim's severance from the firm. C) It is not necessary to notify TJK's clients of Tim's departure, because the advisory will continue to serve its clients as before. D) TJK must notify its clients of Tim's departure within 30 days of Tim's severance from the firm.

An investment adviser firm organized as a general partnership must notify its clients of the departure of a general partner within a reasonable time.

A U.S. citizen owns stock in a Canadian company and receives dividends. The Canadian government withholds 15% of the dividends as a tax. As a result, the investor reports A) a reduction in the investor's ordinary income B) a tax credit on the investor's U.S. tax return C) a tax credit on the investor's Canadian tax return D) a nonrecoverable loss on the investor's U.S. tax return

An investor receives a credit for taxes withheld on investments by countries with which the United States has diplomatic relations; the tax credit directly decreases the investor's American tax liability.

A U.S. citizen owns stock in a Canadian company and receives dividends. The Canadian government withholds 15% of the dividends as a tax. As a result, the investor reports A) a tax credit on the investor's Canadian tax return B) a tax credit on the investor's U.S. tax return C) a nonrecoverable loss on the investor's U.S. tax return D) a reduction in the investor's ordinary income

An investor receives a credit for taxes withheld on investments by countries with which the United States has diplomatic relations; the tax credit directly decreases the investor's American tax liability.

A portfolio that maximizes an investor's preferences with respect to return and risk is called A) the efficient frontier B) an optimal portfolio C) an uncorrelated portfolio D) a diversified portfolio

An optimal portfolio will generally lie on the efficient frontier (which is a graph, not a portfolio). The special nature of an optimal portfolio is that it may not always be the most efficient portfolio (offering the greatest return for the least risk) because it takes into consideration the specific preferences of the individual investor, which might create a bias.

When an agent submits an order ticket to purchase securities for a client, all of the following would appear EXCEPT A) the current market price of the security B) the broker-dealer's name C) the agent's name D) the details of the order

Any order ticket submitted by an agent for execution at a broker-dealer will always include the agent's name and that of the BD. All order details must be listed (e.g. the number of shares, limit or market, etc.), but the current market price is never included.

A broker-dealer holds fully paid-for customer securities for safekeeping. Under the NASAA Statement of Policy on Unethical and Dishonest Business Practices of Broker-Dealers and Agents, the broker-dealer A) must pay interest to the clients B) would be in violation unless a properly executed margin agreement was in effect C) must segregate them D) may lend them to make delivery on short sales

Any securities held in custody by a broker-dealer (or, for that matter, an investment adviser) must be segregated from those belonging to the broker-dealer (or investment adviser). To do otherwise would be to commit the prohibited practice of commingling. Fully paid securities may not be loaned out, only those collateralizing a debit balance may be and, then, only with customer permission.

If interest rates were to decline sharply, which of the following securities is likely to appreciate the most? A) 20-year municipal bond currently trading at par B) 20-year corporate bond currently trading at a small premium C) 20-year mortgage-backed security currently trading at a small discount D) 20-year zero-coupon Treasury bond currently trading at a deep discount

As a rule, the longer the duration, the greater the price appreciation. In this case, all the fixed-income securities have 20-year maturities. Another general rule is that the lower the coupon on the bond, the longer the duration. The zero-coupon bond has the lowest coupon and would likely appreciate the most.

An individual has been employed by a broker-dealer to make cold calls to solicit prospects for the firm's new wrap fee program. Under the USA, it is true to state that this individual A) would be defined as an investment adviser representative B) would be permitted to use the term investment counsel C) does not need supervision because he is only making cold calls D) is not defined as an investment adviser representative because he is only making cold calls

As we know, when a broker-dealer offers wrap fee programs, the exclusion from the definition of investment adviser is lost. Any individual soliciting for that program would be considered an investment adviser representative and would need adequate supervision. Cold calling is about as far as you can get from the role of an investment counsel. U2LO1

If a client wanted an investment that would eliminate interest risk as to principal, you would recommend A) TIPS B) preferred stock C) a bank-insured certificate of deposit D) a 91-day Treasury bill

Because bank-insured CDs are nonnegotiable (we're not discussing the $100k minimum jumbos), there is no market fluctuation caused by changes in interest rates as with marketable securities. If you invest $10,000, you will always get back that $10,000 whenever you cash in the CD, regardless of current interest rates. This is true even when cashing in early. There may be a prepayment penalty, but that is considered separate from interest rate risk. TIPS offer inflation protection and preferred stock is interest rate sensitive in the same manner as a bond. The 91-day T-bill doesn't have much interest rate risk, but if an investor was to attempt to liquidate the holding prior to maturity and interest rates increased, there could be a loss. U19LO6

An investor who chooses to use preferred stock as an income source instead of bonds would potentially incur which of the following risks? 1Loss of principal 2Price volatility of preferred stock is closely related to interest rates 3Preferred stock cannot be traded as readily as bonds 4If the stock is callable, the client's income can be suddenly lowered A) I, II, III, and IV B) I and II C) III and IV D) I, II, and IV

Because bonds have seniority over any equity security, there is a greater risk of loss of principal with preferred stock than with bonds. The price volatility of preferred stocks, like bonds, is impacted by interest rate changes. Unlike bonds, however, preferred stock does not have a maturity date. This means that preferred shares may never return to their par value, as bonds do at maturity date. Because the preferred stock may have a callable feature, the company can redeem its shares anytime after the call protection period (if any) is over. This usually happens when interest rates have declined, so the client whose stock was called will not be able to reinvest the proceeds at the same rate and could, therefore, suffer an unexpected drop in income. Preferred shares, particularly those listed on the exchanges, are generally easier to trade than corporate bonds (and certainly no worse).

Bob, age 60, has invested $17,000 in his nonqualified variable annuity over the years. The total value has reached $26,000. He wishes to withdraw $15,000 to send his son to college. What is his tax consequence on the withdrawal? A) The entire amount is nontaxable. B) The entire amount is taxable. C) $9,000 is taxable; $6,000 is nontaxable. D) $6,500 is nontaxable; $8,500 is taxable.

Because this is a nonqualified plan, the $17,000 invested is after-tax dollars. Under the Tax Code, the taxable portion is considered to be withdrawn first in any lump-sum distribution. Therefore, the first dollars withdrawn are all taxable until the amount of withdrawal meets or exceeds the growth in the account. Because Bob is over 59½, there is no 10% tax penalty on his withdrawals.

Which of the following statements regarding investment theory is not correct? A) In a well-diversified portfolio, diversifiable risk is zero. B) Combining two stocks with a negative correlation coefficient can significantly reduce the portfolio's standard deviation. C) A correlation coefficient of 0.14 between the returns of Stock C and Stock L indicates that very little of Stock C's returns can be attributed to the returns of Stock L. D) The beta coefficient may be used to help select a portfolio that is consistent with an investor's willingness to assume unsystematic risk.

Beta is a measure of systematic risk, not unsystematic risk. The beta coefficient may be used to help select a portfolio that is consistent with an investor's willingness to assume systematic risk. Diversifiable risk (unsystematic risk) can be brought down to zero with proper diversification. Including securities with negative correlation is a prime method of reducing overall risk (expressed by the portfolio's standard deviation) and the closer the correlation coefficient gets to zero (and 0.14 is close), the more random the relationship between the returns earned by two securities.

Your client with $100,000 to invest is looking for maximum current income. Which of the following would offer the highest current return? A) $200,000 of utility common stock paying a current dividend of 3.5% B) $100,000 of zero-coupon bonds with a yield to maturity of 6% C) $100,000 AA-rated corporate bonds trading at par with a 6% coupon rate D) $100,000 market value of corporate bonds selling at a premium and yielding 6% to maturity

Bonds selling at a premium have higher coupons than those selling at par. Therefore, the current yield on those bonds is higher than the ones at par, even though they would have the same yield to maturity. The zero-coupon bonds offer no current income and the investor only has $100,000 to invest, so the utility stock is not a viable option.

Which of the following statements correctly describe similarities between exchange-traded funds and closed-end investment companies? 1.There are a limited number of outstanding shares. 2They are traded on registered stock exchanges. 3They trade at prices that are not dependent upon but close to their net asset value. 4Investors pay commissions to purchase and liquidate their positions. A) I and IV B) II and III C) I and III D) II and IV

Both exchange-traded funds and closed-end investment companies are traded on exchanges; therefore, investors pay a commission when purchasing and liquidating shares. Only closed-end investment companies have a limited number of shares. Closed-end funds may trade at significant premiums or discounts from their NAV, while ETFs rarely stray far from NAV.

A pooled investment with a share price generally different from its net asset value (NAV) per share is most likely A) an index fund. B) an exchange-traded fund. C) a closed-end fund. D) an open-end fund.

Closed-end funds' share prices can differ significantly from their NAVs. Open-end fund shares are purchased and redeemed based on their NAVs. Market forces keep exchange-traded fund share prices close to their NAVs because arbitrageurs can profit by trading when there are differences. U14LO3

One year ago, ABC Widgets, Inc., funded an expansion to its manufacturing facilities by issuing a 20-year first mortgage bond. The bond is secured by the new building and land. The bond was issued with a 5.5% coupon and is currently rated Aa. The current market price of the bond is 105 resulting in a current yield of approximately: A)5.24%. B) 5.61%. C) 4.99%. D) 5.50%.

Corporate bonds are quoted as a percentage of the $1,000 par value. A market price of 105 is equal to $1,050 (105% × $1,000). Each $1,000 5.5% bond pays $55 of interest annually ($1,000 × 5.5% = $55.00). Current yield equals the annual interest divided by the current price of $1,050. The calculation is $55 ÷ $1,050, which is equal to approximately 5.24%. Because the bond is at a premium, the current yield must be below the nominal yield, which removes two of the choices from consideration.

During the previous fiscal year, The Kaplan Family Trust received $24,000 in dividends and $35,000 in interest from corporate bonds. Securities transactions during the year resulted in long-term capital gains of $48,000, $20,000 of which were reinvested in the corpus. The DNI for the Kaplan Family Trust is A) $79,000 B) $107,000 C) $87,000 D) $11,000

Distributable Net Income (DNI) is dividends and interest plus capital gains that have not been reinvested back into the trust. In this case, $24,000 + $35,000 + $28,000 = $87,000.

If your 60-year-old customer purchases a nonqualified variable annuity and withdraws some of her funds before the contract is annuitized, what are the consequences of this action? A) Capital gains tax on earnings exceeding basis B) 10% penalty plus payment of ordinary income tax on all funds withdrawn C) Ordinary income tax on earnings exceeding basis D) 10% penalty plus payment of ordinary income tax on all funds withdrawn exceeding basis

Distributions from a nonqualified plan represent both a return of the original investment made in the plan with after-tax dollars (a nontaxable return of capital) and the income from that investment. The income was deferred from tax over the plan's life, so it is taxable as ordinary income once distributed. A 10% penalty applies only if distributions begin before age 59½.

If a company with 10 million shares outstanding with total earnings of $50 million pays a $2 dividend, the dividend payout ratio is A) 25% B) 40% C) 20% D) 4%

Dividend payout ratio is determined by dividends paid per share divided by earnings per share. In this case, earnings per share (EPS) is $50 million ÷ 10 million shares = $5 per share. The company paid out in dividends $2 for each $5 earned for a 40% payout ratio ($2 ÷ $5).

An investor owns five DEF call options with a strike price of $40. The options are European style. If the holder exercises, the cost will be A) $20,000. B) zero because European options are exercisable only at expiration. C) $2,000. D) $4,000.

Each option contract represents 100 shares. Exercising five call options means buying 500 shares at a price of $40 each, which equals $20,000. Although it is true that European-style options are exercisable only at expiration, nothing in the question indicates the investor tried to exercise before then.

Which of the following is the least suitable mutual fund transaction? A) Encouraging a retired 65-year-old investor to invest a small percentage of his savings in a large-cap growth fund B) Encouraging an investor in a high tax bracket with an income objective to invest in a municipal bond fund C) Encouraging an investor in his early 30s to invest in an emerging markets mutual fund D) Encouraging a mutual fund shareholder to switch from one fund family to another while a deferred load is in existence

Encouraging a mutual fund shareholder to switch from one fund family to another while a deferred load is in existence is not in the client's best interest, because the client might be subject to substantial additional sales charges.

A customer's portfolio has a beta coefficient of 1.1. If the overall market increases by 10%, the portfolio's value is likely to A) decrease by 10% B) decrease by 11% C) increase by 10% D) increase by 11%

Explanation A beta of 1.1 means the portfolio is considered to be 1.1 times more volatile than the overall market. If the market is up 10%, the portfolio with a beta of 1.1 is likely to be up 11%.

As used in the regulations, the term impersonal investment advice means A) investment advisory services provided strictly by subscription B) investment advisory services provided where the client does not know the identity of the investment adviser representative C) investment advisory services provided by a team of advisers D) investment advisory services provided by means of written material or oral statements that do not purport to meet the objectives or needs of specific individuals or accounts

Explanation This is the way the term is defined.

In contrast with a typical forwards contract, futures contracts have: A) standardized terms. B) less liquidity. C) greater counterparty risk. D) nonstandard terms.

Futures are contracts that trade on exchanges and have standardized terms, in contrast with forwards contracts, which are customized instruments. A futures clearinghouse reduces counterparty risk by guaranteeing the performance of buyers and sellers. Because futures contracts trade on organized exchanges and have standardized terms, they are more liquid than forwards contracts.

An investor who is long XYZ stock would consider going long an XYZ call to A) protect against an increase in the market price of XYZ stock B) obtain income from the premium C) protect against a decrease in the market price of XYZ stock D) hedge the long position

Going long a call means that you have bought it. Only sellers of options generate income. If you wish to hedge your long stock position, you buy a put, not a call. That leaves us with two choices that are polar opposites. Good test-taking skills teach us that, in almost all cases, when we see that, one of those must be the right answer. Buying a call is bullish. Forget the first part (you are long the stock). You would buy a call so that, if the price of the stock went up, you could exercise at the lower strike price of your call option.

Based on the following information, which stock is most likely to appeal to a growth investor? A) Dividend yield of 0.3% B) Dividend payout ratio of 65% C) Book value of $22 per share, current market value of $17 per share D) P/E ratio of 8:1

Growth investors usually seek stocks with high-growth expectations, reflected by a higher-than-normal P/E ratio, typically 20:1 or higher, and a low dividend yield, usually caused by a low dividend payout ratio. It would be unlikely to find a growth stock selling for close to its book value and certainly not below it.

The SEC has enumerated specific items that must be included in Investment Adviser written compliance manuals EXCEPT A) the advisory firm must review policies and procedures at least on an annual basis B) the advisory firm should implement procedures for allocating investment opportunities such as best executions among clients C) the advisory firm should indicate the educational requirements necessary for employment D) the advisory firm must monitor the consistency of portfolios with guidelines established by clients, disclosures, and regulatory requirements

Guidelines under SEC rules require (at minimum) that the chief compliance officer of each federal covered investment adviser conduct an annual review of its compliance procedures. Among the duties of the compliance officer is to monitor the consistency of portfolios with guidelines established by clients, disclosures, and regulatory requirements. The firm should implement procedures for allocating investment opportunities such as best executions among clients. If the firm does have internal educational requirements, that would be found in its HR manual, not in its compliance manual.

When compared to mutual funds, which of the following statements regarding hedge funds is least accurate? Hedge funds A) are generally only appropriate for qualified investors. B) use derivatives to a greater extent. C) can take both long and short positions. D) tend to be more diversified in order to hedge risk.

Hedge funds portfolios are more concentrated (i.e., less diversified), so that individual positions provide a significant contribution to the portfolio's return. In most cases, only accredited investors may invest in hedge funds. A major difference between hedge funds and mutual funds is the ability of the hedge fund to take short positions and one way the hedge fund obtains greater leverage is through the use of derivatives.

Which of the following statements regarding unsolicited orders is TRUE? A) Unsolicited orders are nonexempt transactions under the USA. B) A client may purchase, at his own initiative, securities trading in the secondary market through an agent who otherwise is prohibited from soliciting the order. C) The state Administrator may not require the client to sign an acknowledgment that the order was unsolicited. D) The state Administrator may not prohibit the solicitation of specific securities in the state.

If a client requests the purchase of a security that an agent is prohibited from soliciting, the agent can accept the order and mark the order unsolicited. This is the most common of the exempt transactions.

In the event that a filing with the state securities Administrator is found to have material misstatements or omissions, a correcting amendment must be filed? PROMPTLY

If a filing with the Administrator is found to have material misstatements or omissions, an amendment must be filed promptly with the office of the Administrator.

Customer A and Customer B each have an open account in a mutual fund that charges a front-end load. Customer A has decided to receive all distributions in cash, while Customer B automatically reinvests all distributions. How do their decisions affect their investments? Receiving cash distributions may reduce Customer A's proportional interest in the fund. Customer A may use the cash distributions to purchase shares later at NAV. Customer B's reinvestments purchase additional shares at NAV rather than at the offering price. Due to compounding, Customer B's principal will be at greater risk. A) II and IV B) II and III C) I and III D) I and IV

If the customer elects to receive distributions in cash while other investors purchase shares through reinvestment, his proportional interest in the fund will decline. Automatic reinvestment is always at NAV.

A customer and his spouse own shares in the ABC Fund as joint tenants with rights of survivorship. If the customer dies, what happens to the shares in the account? A) Ownership of the shares must be determined by probate court. B) Half the shares would belong to the spouse, and the remaining half would be distributed to the customer's estate. C) The account would be frozen until the estate was settled. D) The spouse would own all the shares.

In a JTWROS account, securities pass to the surviving owner. The account does not have to be frozen but can continue to enter orders.

in a scheduled premium variable life insurance policy, all of the following are guaranteed EXCEPT A) the ability to borrow at least 75% of the cash value after the policy has been in force at least 3 years B) a minimum death benefit C) the right to exchange the policy for a permanent form of insurance, regardless of health, within the first 24 months D) a minimum cash value

In a variable life insurance policy, a minimum death benefit is guaranteed, but no cash value is guaranteed. There is a contract exchange privilege during the first 24 months allowing the conversion of the variable policy to a comparable form of permanent insurance and the 75% cash value loan minimum applies after the 3rd year of coverage.

Ebony sets up a revocable trust, naming her daughter, Sylvia, as the sole beneficiary. Ebony has appointed the Pacific Atlantic Trust Institution (PATI) as the trustee. Any income to the trust will be taxable to A) the beneficiary B) the grantor C) the trustee D) the trust

In almost all cases, income received into a revocable (grantor) trust, whether distributed or not, is taxable to the grantor. Things are different when the trust is irrevocable, but much more complicated and not likely to be tested.

Under the USA, all of the following are exempt transactions EXCEPT A) a sale of a primary offering registered with the SEC B) isolated nonissuer transaction C) unsolicited customer orders D) transaction executed by a trustee in bankruptcy

In almost every instance, an issuer transaction—that is, one for the benefit of the issuer—will not be considered an exempt transaction. Exempt transactions include isolated nonissuer transactions; transactions between an issuer and an underwriter; transactions by an executor, Administrator, sheriff, marshal, trustee in bankruptcy, guardian, or conservator; any sale or offer to a bank, savings institution, investment company, or other financial institution; and private placements.

Which of the following bonds would be the least price sensitive to changes in market interest rates? A) 10% BB bond due in 21 years with a YTM of 8.7% B) 6% AA bond due in 18 years with a YTM of 6.8% C) 4.5% Treasury bond due in 20 years with a YTM of 4.1% D) Zero due in one year with a YTM of 6%

In almost every question like this, the zero will have the longest duration and the greatest price sensitivity to interest rate changes. This is the odd case where the zero is due so soon that its duration is by far the shortest of any of the choices. Shorter duration means less price sensitivity.

ABC Furniture Company wishes to raise capital by issuing some securities in its home state. The CEO of the company feels that registration with the Administrator is unnecessary because the issue is exempt. Should ABC be ordered to appear at a hearing, the burden of proving its issue is exempt is on A) the CEO B) the Administrator C) the hearing panel D) the company

In any case where there is a question as to the legality of a specific exemption, the burden of proof is always on the party requesting the exemption.

Under NASAA's Statement of Policy on Dishonest or Unethical Business Practices of Broker-Dealers and Agents, what factor is NOT considered in determining whether excessive trading has occurred? A) The length of time the account has been opened B) The financial condition of the account and the financial resources and goals of the client C) The frequency of trading D) The amount of trading Explanation

In determining whether excessive trading has occurred, the length of time the account has been opened would not be factor. Consideration must be given to the amounts and frequency of trading in view of the financial resources, investment objectives, and character of the client's account. All factors must be considered together and not individually. Frequent trading and trading large amounts are not wrong in and of themselves; they are permitted if suitable for that particular customer.

Suzie McQueen has a very successful interior design shop she has run as a sole proprietorship. She has just celebrated her 60th birthday and has been giving thought to an eventual sale of the business. She wants your opinion on whether she should incorporate or change to a partnership. You might respond that A) the corporate form of business structure would be the least expensive to form B) the corporate form of business structure would be the easiest for ultimate transfer of ownership C) the partnership form of business structure would enable Suzie to maximize her sale price D) the partnership form of business structure would be the easiest for ultimate transfer of ownership

In general, the corporate form of business leads to the easiest transfer of ownership. Because Suzie would probably own 100% of the stock, all she would have to do is sell that stock to a new purchaser and the corporation could continue just as before. If Suzie wanted to reorganize as a partnership, she would have to bring in at least one additional individual, ending her total ownership of the business. Even then, a partnership interest is not as easy to sell as stock.

In the Howey decision, the U.S. Supreme Court held that in order for an investment contract to be considered a security, it must represent A) personal interest in a business B) an investment of money in a common enterprise with the expectation of profit from the managerial efforts of others C) debt in a publicly traded corporation whose managers are engaged in commercial activity D) an investment of money in a common enterprise with the expectation of profit from the efforts of the investor

In the Howey decision, the U.S. Supreme Court held that a security must represent an investment of money in a common enterprise with the expectation of profit from the managerial efforts of others.

Which of the following is not included in the definition of broker-dealer as found in the Uniform Securities Act? A) Investment advisers B) Attorneys C) Credit unions D) Banks

In the Uniform Securities Act, it specifically states: "Broker-dealer" means any person engaged in the business of effecting transactions in securities for the account of others or for his own account. "Broker-dealer" does not include (1) an agent, (2) an issuer, (3) a bank, savings institution, or trust company. Attorneys are excluded from the definition of investment adviser, as long as their advice is incidental to their legal practice, but that exclusion does not apply to the term "broker-dealer". Even though credit unions engage in banking activity, they are not included in the exclusion. Being an investment adviser does not exclude a person from the need to register as a broker-dealer if that person is performing the functions of a BD. Banks are excluded.

An agent mistakenly sold an unregistered, nonexempt security to a customer. Which of the following actions should the broker-dealer take? 1Offer to buy the security back from the customer 2Ask the customer to sign a customer agreement 3Register the stock by notification 4Offer to pay interest at an annual rate determined by the Administrator, less income paid, from the date the security was purchased A) II and III B) I and III C) I and IV D) I, II, III, and IV

In the case of an agent who mistakenly sells an unregistered, nonexempt security, the broker-dealer should offer to buy back the security from the customer and pay the customer interest on the amount invested in the security for the period from the original purchase to the resale back to the firm, minus any income or profit realized by the client on the security. This is known as the right of recession

An IAR with a state-registered adviser would like to employ the services of an individual as a solicitor to help bring in more business. The solicitor will be compensated by receiving a percentage on all assets placed under management. In order to do this, all of the following must be complied with EXCEPT A) the solicitor must be registered as an IAR in order to receive compensation based upon advice B) the IA and the solicitor brochure must be delivered at least 48 hours before entering into the contract C) the terms of the compensation must be spelled out D) disclosure of the arrangement must be made to the client

In this example, the IA brochure and solicitor brochure are required to be delivered with the sales presentation. The contract is not signed until the client agrees to engage the services of the IA If these are not delivered at least 48 hours prior to signing, the client has a 5-day penalty-free withdrawal option.

Each of the following terms is commonly found in modern portfolio theory EXCEPT A) the capital asset pricing model B) the efficient set C) the feasible set D) the internal rate of return

Internal rate of return (IRR) is not a component of modern portfolio theory as are the other 3 terms. (TIME VALUE)

High-yield bonds are frequently called junk bonds. Which of the following expresses the highest rating that would apply to a junk bond? A) BBB B) BB C) CCC D) CC

Investment-grade bonds run from a highest Standard and Poor's rating of AAA (Aaa − Moody's) down to BBB (Baa − Moody's). When the rating gets to BB (or Ba) the bond is considered high yield, or a junk bond. U13LO4

The distributable net income (DNI) of a simple trust would not include A) realized capital gains. B) interest received on corporate bonds. C) interest received on municipal bonds. D) dividends received.

It is capital gains that are reinvested in the corpus (body) of a simple trust which are not part of DNI. Although the interest on municipal bonds in not taxable, it is still included as part of the DNI

A working group convened by NASAA has developed a model fee disclosure schedule to help investors better understand the costs involved in doing business with their broker-dealer. The template has broker-dealers disclosing which of the following fees? A) Markups and markdowns B) Commissions C) Account closing fees D) Advisory fees Explanation

It is very common for a broker-dealer to charge a fee for processing the closing of an account. There are 3 primary expenses involved with brokerage accounts that are not included in the fee disclosure template. Those are: commissions; markups and markdowns; and advisory fees for those firms that are also registered as investment advisers.

A policyowner could surrender a whole life insurance policy and choose from all the following EXCEPT A) transferring the policy to another person B) purchasing an extended term life policy C) purchasing a reduced coverage whole life policy D) taking the cash value

Life insurance policies are nontransferrable. Upon surrender, the cash value may be taken or used to purchase extended term insurance or a reduced value, paid-up, whole life policy.

NASAA's Model Rule on Unethical Business Practices of Investment Advisers, Investment Adviser Representatives, and Federal Covered Advisers would consider the adviser to be engaging in an unethical business practice if he loaned money to a client other than one A) who was an immediate family member of the adviser B) who was an affiliate of the adviser C) who was in the money-lending business D) borrowing under the same terms and conditions as the client could find at a commercial bank

Loaning money to a client is prohibited unless the investment adviser is a financial institution engaged in the business of loaning funds or the client is an affiliate of the IA. Please note that because this question deals with an IA lending money, the fact that the IA's client is in the money-lending business is of no consequence. That would only be an issue if the question dealt with the IA borrowing money.

As appropriate to the scale and complexity of a firm's business, elements of an effective practice framework for managing conflicts of interest include all of the following EXCEPT A) ensuring that the firm remains solvent for protection of customers and employees alike B) avoiding severe conflicts, even if that avoidance means foregoing an otherwise attractive business opportunity C) establishing mechanisms to identify conflicts in a firm's business as it evolves D) training staff to identify and manage conflicts in accordance with firm policies and procedures

Managing conflicts of interest does not take into consideration making enough money to remain solvent.

With an annuity, 1.taxes on earned dividends, interest, and capital gains are paid annually until the owner withdraws money from the contract. 2. random withdrawals are taxed on a LIFO basis. 3. invested in a nonqualified annuity represents the investor's cost basis. 4. upon withdrawal, the amount exceeding the investor's cost basis is taxed as ordinary income. A) I only B) I, II and IV C) II, III, and IV D) IV only

Money randomly withdrawn (not annuitized) is handled under LIFO tax rules. Money invested in an annuity represents the investor's cost basis and on withdrawal, the amount exceeding the investor's cost basis is taxed as ordinary income. Taxes on earned dividends, interest, and capital gains are not paid annually. They are deferred and paid later, when the owner withdraws money from the contract.

A client of yours owns some convertible preferred stock. She notices an article in the business section of her local newspaper that reports the company is going to pay a 20% stock dividend on their common stock. She wants to know how this will affect her? A) More than likely, the price of the preferred stock will rise. B) She will also receive 20% more shares because preferred stock has a priority claim ahead of common. C) There will be no effect. D) If there is an antidilution clause, her conversion privilege will permit her to acquire 20% more shares than before the stock dividend.

Most convertible securities are sold with antidilutive clauses that provide for an adjustment in the number of shares based on stock splits or stock dividends.

The following table shows the individual weightings and expected returns for the 3 stocks in an investor's portfolio: StockWeightE(Rx)V0.4012%M0.358%S0.255% What is the probable return of this portfolio? A) 9.55% B) 8.33% C) 9.05% D) 8.85

Multiplying the weight of each asset by its expected return, then summing, produces the following: E(RP) = 0.40(12) + 0.35(8) + 0.25(5) = 8.85%.

It would not be considered an unethical and dishonest business practice for an agent registered with a broker-dealer to divide or otherwise split the agent's commissions, profits, or other compensation from the purchase or sale of securities 1.with any person also registered as an agent for the same broker-dealer 2.with any person also registered as an agent for a broker-dealer under direct or indirect common control 3.as long as the arrangement is in writing 4.as long as the client has approved of the sharing arrangement A) III and IV B) I, II, III, and IV C) I, II, and III D) I and II

NASAA's Statement of Policy on Unethical or Dishonest Business Practices of Broker-Dealers and Agents permits commission sharing as long as the agents are properly registered with the same broker-dealer or one under common control. There is no requirement for the arrangement to be in writing, and the customer has no say so in this matter.

A customer of an investment adviser inadvertently mails some stock certificates to the IA. The IA does not maintain custody of customer assets. If the certificates were received on a Monday, NASAA rules would requires that the certificates be A) forwarded to the broker-dealer promptly B) returned the same day C) returned no later than Tuesday D) returned no later than Thursday

NASAA's custody rules require that an investment adviser who does NOT maintain custody must return certificates that are mistakenly sent within 3 business days. When it comes to checks, it depends on how the check is drawn. If made out to the investment adviser, it must be returned; if made out to a third party (usually the executing broker-dealer), it must be forwarded to that third party. In either case, the time limit is 3 business days (might be shown as 72 hours on the exam).

Which of the following statements is NOT correct? A) Net present value (NPV) is the difference between the initial cash outflow (investment) and the future value of discounted cash flows. B) Time-weighted returns show performance without the influences of additional investor deposits or withdrawals from the account. C) Net present value analysis (NPV) is a commonly used time value of money technique employed by businesses and investors to evaluate the cash flows associated with capital projects and capital expenditures. D) Internal rate of return (IRR) is a method of determining the exact discount rate to equalize cash inflows and outflows, thus allowing comparison of rates of return on alternative investments of unequal size and investment amounts.

Net present value (NPV) is the difference between the initial cash outflow (investment) and the present value of discounted cash flows (NPV = PV of CF − cost of investment). That is why it is called net present value instead of net future value

An agent may determine which securities to purchase or sell for a client when A) written discretion authority has been received by the broker-dealer within 10 days of the initial discretionary transaction B) written or oral discretion authority has been received by the broker-dealer within 10 days of the initial discretionary transaction C) written or oral discretion authority has been received by the broker-dealer before executing the first discretionary transaction D) written discretion authority has been received by the broker-dealer before executing the first discretionary transaction

No broker-dealer or any of its employees shall exercise any discretionary power in any customer's account or accept orders for an account from a person other than the customer without first obtaining written authorization from the customer. It is an investment adviser who may act with oral consent for a period of 10 days from the initial discretionary trade.

Which of the following statements regarding agent registration under the Uniform Securities Act are TRUE? 1. In the absence of any action by the Administrator, the effective date of a registration is noon of the 30th day. 2. The Administrator may initiate a disciplinary action within 2 years of an agent's withdrawal of registration. 3. The administrator may request the agent furnish a statement of assets and liabilities. 4. If, before the effective date of the registration, the Administrator requires amendments to the application, the registration will be considered to have first been filed upon filing of those amendments. A) I and IV B) III and IV C) II and III D) I and II

Normally, registration of persons becomes effective at noon of the 30th day following filing. If the Administrator requires the filing of amendments, the clock starts over again with the filing of those amendments. Agents do not have financial requirements, and the Administrator has a maximum of 1 year after termination to initiate any actions.

As an incentive to encourage clients to invest in a particular stock recommended by the broker-dealer, clients are told that anytime within 6 months after the purchase date, they may sell the stock back to the firm at original cost plus interest at the state's legal rate. This would be A) an offer that could only be made to accredited investors B) a violation of the antifraud provisions of the Uniform Securities Act C) a right of rescission D) a prohibited guarantee against loss

Offering to buy back a stock at its original cost, even without paying interest, is a prohibited guarantee against loss. Rescission is only when there was something improper about the sale. Technically, this offer is not a case of fraud and, in any event, we must always select the answer that best addresses the question—in this case, a guaranteed price.

A 45-year-old investor takes a lump-sum distribution from a nonqualified variable annuity. How is the distribution taxed? 1The entire amount is taxed as ordinary income. 2The growth portion is taxed as ordinary income. 3The growth portion is taxed as a capital gain. 4The growth portion is subject to a 10% penalty. A) I and IV B) II and IV C) III and IV D) II and III

On withdrawals from a nonqualified annuity, taxes are paid only on the amount that exceeds cost basis (the amount paid into the annuity). In this case, the investor is taking a lump-sum distribution before reaching age 59½ and must pay an additional 10% penalty on the taxable amount

Parsimonious Planning Associates (PPA), an investment adviser with over $250 million in assets under management, is accused of violating the antifraud provisions of the Uniform Securities Act. Which of the following statements is true? A) The Administrator of the state where PPA's principal office is located is the only person authorized to investigate the charge. B) The Administrator of the state where the alleged fraud occurred may investigate the charge. C) Because PPA is a federal covered adviser, only the SEC has the jurisdiction to investigate the charge. D) No investigation may take place until the charges are proven true.

Once an investment adviser's AUM reaches $110 million, registration with the SEC is required. That makes PPA a federal covered adviser. Although covered advisers are exempt from the jurisdiction of the state for most things, one area in which they are not is when the antifraud provisions of the USA are breached. In that case, jurisdiction will usually rest with the Administrator of the state where the alleged fraudulent activity took place. In some states, the Administrator will refer the charges to the Administrator of the state where the IA's principal office is located, but that is not mandatory. Because the violation is of the USA, the SEC has no jurisdiction. Without an investigation, how can the charges be proven true (or false)?

Which of the following statements regarding secondary trading in the private equity market is TRUE? A) A trade in a secondary market may be motivated by the desire for increased access to deals in the primary market. B) Secondary markets are a form of distressed securities markets wherein limited partners sell securities with troubled performance histories. C) Secondary trading generally causes investors to have to wait over a longer time period to generate returns from their private equity investment. D) Secondary trading makes it more difficult for investors to make strategic shifts in the private equity allocations within their portfolios.

One of the advantages to secondary trading is that making secondary purchases can give an investor exposure to a general partner, which can create opportunities to gain access to future opportunities from that partner. The other statements are all incorrect. Secondary trading may allow investors to get into a private equity deal at a later stage and, thus, realize positive returns more quickly. Secondary trading provides liquidity and makes it easier for investors to make strategic shifts in their portfolios. Secondary markets are often used by investors due to changing portfolio needs, rather than a change in the value of their private equity funds. U14LO5

A terminally ill client wishing to access a portion of the cash value in his whole life insurance policy while still providing a death benefit for his beneficiaries could do so by A) taking out a policy loan B) selling the policy in a viatical settlement C) surrendering the policy for its cash value D) converting it into a term policy

One of the benefits of whole life insurance is the ability to borrow against the guaranteed cash value in the policy. At death, the amount of the loan is paid off from the death benefit, but the remainder is then paid to the beneficiaries of the policy. Surrendering the policy cancels the death benefit, and the purchaser of the viatical is now the one who determines the beneficiaries. You can't convert permanent insurance to term (and the exam will not consider the situation of leaving the cash value to purchase extended term insurance, which wouldn't work here anyway).

A form of business structure that exposes all personal assets of the owner to creditors is A) the C corporation B) the limited partnerships C) the LLC D) the sole proprietorship

One of the reasons why few large businesses are organized as sole proprietorships is the fact that all personal assets, not just those of the business, are placed at risk if the business fails. In each of the other choices, the maximum potential loss is the amount of the investment.

One of the rights of those owning common stock is the opportunity to vote on issues brought up at the corporation's annual meeting. To be eligible to cast a vote, A) the company must be current on its dividends to preferred stockholders B) ownership must be established by the record date C) the stock must be paid for in full before the annual meeting D) the stockholder must be a natural person

Only stockholders who are on the company's books by the record date are eligible to vote.

Which of the following statements are generally TRUE of the buy-and-hold strategy? Equities would grow relative to fixed income Lower taxes and transactional costs Easy to manage The portfolio would more accurately demonstrate the client's investment objectives and risk tolerance A) III and IV B) I, II, and III C) II, III, and IV D) I and II

Over the long run, using the buy-and-hold strategy with equity securities has outperformed the rate of return on fixed income investments. With few transactions, there are almost no commissions and capital gains taxes. Of all strategies, this is the easiest to follow. There is no way to determine the client's objectives or risk tolerance based on the decision to buy and hold. The portfolio might contain small-cap stocks or large-cap stocks. It might contain 90% equities or 75% debt securities. Investors with differing goals and risk tolerance can use this strategy.

John, an investment adviser, employs an investment adviser representative who is found guilty of defrauding many of the firm's clients over a long period of time. Which of the following is (are) TRUE under the Uniform Securities Act? I. The investment adviser representative is subject to criminal penalties specified in the act and to civil liabilities resulting from clients who sue as a result. II. John may be subject to civil liabilities resulting from actions taken by the investment adviser representative. III. John is not subject to civil liabilities as a supervisor if he can prove that he had no knowledge of the actions of the representative and, while exercising reasonable care, he could not have had knowledge of the violations.

Persons convicted of willful violations are subject to the criminal penalties specified in the act (3 years and/or $5,000 fine). Thus, the representative is subject to criminal penalties and civil liabilities resulting from clients who sue him for loss of money. The act subjects to civil liabilities any person supervising those who violate the law. However, a supervisor may not be held liable for the actions of those whom he supervises if it can be proved that the supervisor used reasonable care to discover and prevent the violations and has no knowledge of the violations.

Which of the following would NOT be considered an unethical practice for a registered investment adviser? A) Acting as a principal in a recommended transaction without consent of the client prior to completion of the trade B) Failing to notify the Administrator that the adviser is maintaining custody of client funds and securities C) Unfairly criticizing an estate plan prepared by the client's attorney D) Acting as a trustee for a client's trust

Please notice the word not in the question. Although acting as a trustee for the client's trust is probably not a good business practice, it is not included in the list of unethical activities for an adviser.

Present value is a computation frequently used to determine the amount of deposit needed now to meet a future need, such as a college education. If an investor uses an expected return of 8%, but the actual return over the period is 6%, A) the accumulated value will meet the objectives B) the yield to maturity will be lower than anticipated C) the future value will not be able to be computed D) the present value was insufficient to meet the objective

Present value is the amount deposited to meet a future goal based on an expected rate of return. If the return is lower than expected, the amount deposited will not grow to the required amount (a bad thing).

All of the following would flow through as a loss to limited partners except A) depletion. B) principal repayment on partnership debt. C) accelerated depreciation. D) interest payments on partnership debt.

Principal repayments are not an expense for tax purposes. The interest on the debt is an expense and, along with depletion and depreciation expense, does flow through to the limited partners as passive loss.

Which of the following would be excluded from the definition of investment adviser under the Uniform Securities Act? A) A finance teacher at a local community college who offers weekend seminars on comprehensive financial planning at a very reasonable price B) A broker-dealer charging a separate fee for investment advice C) The publisher of a weekly newsmagazine, sold on newsstands, that contains at least 5 stock recommendations per issue D) A civil damages attorney who advertises that he is available to assist clients in suggesting appropriate investments for their successful claims

Publishers of general circulation newspapers and magazines are excluded from the definition of investment adviser. A broker-dealer loses its exclusion the moment it offers advice for a separate charge, as does an attorney who holds himself out as offering investment advice. Normally, a teacher is excluded, but not when charging for advice, as would appear to be the case here. On this examination, the term "comprehensive financial planning" always includes securities advice.

It would be least likely for dividends paid on which of the following investments to meet the requirements to be considered qualified? A) REITs B) Equity mutual funds C) Common stock D) Preferred stock

Qualified dividends are those eligible for the reduced income tax rates. Those rates can be as low as 0% and as high as 23.8%, with most falling within the 15% or 20% bracket. We don't expect the exam to test on the requirements for a dividend to be considered qualified or how you reach that 23.8% rate. You may be asked about REITs, and their dividends do not meet the definition. In similar fashion, dividends on bond funds and money market funds are not qualified because the majority of those dividends represent interest earned by the fund.

Washington, Adams, and Jefferson, Inc. (WAJI) is an investment adviser whose principal and only office is in Alexandria, VA. WAJI's sole business is advising institutional investors. Rutherford Buchanan is employed by the firm in the main office and has the responsibility of servicing the firm's bank and insurance company clients. Which of the following statements is correct regarding Rutherford's licensing requirements? A) Rutherford is exempt from registration because he has fewer than 6 retail clients. B) Rutherford must register as an IAR of WAJI with the state of Virginia. C) Rutherford cannot register as an IAR of WAJI because providing advice exclusively to institutions exempts the firm from registration. D) Rutherford is exempt from registration because his only clients are institutions.

Regardless of whom the clients are, Rutherford has a place of business in Virginia and that requires registration with the Administrator as an IAR. If WAJI does business in other states where it does not have a place of business, it is exempt from registration because the only clients are institutions. If WAJI is not registered in the state, Rutherford can't register as their IAR. The de minimis exemption for fewer than 6 retail clients only applies when there is no place of business in the state.

All of the following statements regarding scheduled premium variable life insurance are correct EXCEPT A) better than anticipated results in the separate account could lead to a reduction in annual premium B) the policyowner has the right to change the selection of subaccounts C) once selected, the policyowner may change payment modes D) premiums are determined based on age and sex of the insured

Scheduled (fixed) premium variable life premiums are fixed. It is universal life that has flexible premiums.

Under the provisions of the Uniform Securities Act, securities exempt from registration requirements include 1securities issued by the U.S. government 2securities issued by a building and loan association organized under the laws of any state and authorized to do business in this state 3bonds issued by an insurance company organized under the laws of any state and authorized to do business in this state A) I and II B) I, II, and III C) I only D) II and III

Securities exempt from registration requirements include securities issued by the state or U.S. government; securities issued by foreign governments with whom the U.S. maintains diplomatic relations; and any securities issued by savings and loan or building and loan associations, insurance companies, and credit unions authorized to do business in this state.

Differences between static and interactive content on social media include 1only static content can be reused by others 2only static content needs preapproval 3only static content can be changed by the person who originated it 4only interactive content can be commented on by others A) II and IV B) II and III C) I and IV D) I and III

Static content requires preapproval. Interactive content can be reused by others and can be commented on by others. Both static and interactive content can be changed by its originator, but static can only be changed by its originator and interactive by the originator or others.

A 74-year-old widower has been your client since his early 50s. He is a well-informed investor and has always seemed capable of understanding most investment concepts you have presented. At least twice a year, the 2 of you meet to evaluate his current financial situation and objectives. In your last meeting, it seemed to you that he was distracted and somewhat forgetful. It would be appropriate for you to do all of the following EXCEPT A) take detailed notes on future conversations and meetings with him B) inform your supervisor of your concerns about his memory loss C) ask him to invite a friend or family member to accompany him to appointments with you D) wait to see if there are further causes for concern about his capabilities

Taking action in advance could help protect you and your firm should a client subsequently indicate that he does not remember having agreed to a recommendation. Taking detailed notes can help verify what has been discussed in conversations or at meetings. Having others present may help to verify what has been discussed and agreed upon.

f a technician believed in the importance of volume, which of the following would indicate bullish sentiment? A) Prices increase on heavy volume. B) Prices decrease on light volume. C) Prices increase on light volume. D) Prices decrease on heavy volume.

Technicians watch volume changes along with price movements as an indicator of changes in supply and demand. A price increase on heavy volume relative to the stock's normal trading volume is interpreted as an indication of bullish activity.

Maria, age 49, was discussing with some coworkers the recent family vacation she took. She commented that she was able to afford it by taking a penalty-free withdrawal from her retirement plan. Based on that statement, Maria must be covered under A) a 403(b) plan. B) a defined benefit plan. C) a 401(k) plan. D) a 457 plan.

The 457 plan is unique in that it is the only tax-qualified retirement plan permitting withdrawals, for any reason, before reaching 59½ without penalty. All qualified plans have exceptions to the 10% penalty tax, but only the 457 allows the withdrawals for any reason. Even though there is no early distribution tax, Maria will still owe ordinary income tax on the amount withdrawn - the 457 benefit is only that there is no additional 10% tax.

An investor purchases a single premium deferred index annuity with a 6% bonus feature. The premium was $100,000. The annuity has an 80% participation rate with a 10% cap. If the underlying index increased by 15%, the account's value at the end of the year would be closest to A) $118,720. B) $116,000. C) $116,600. D) $110,000.

The 6% bonus means that the client's initial payment is increased by 6%. That means the account shows a starting balance of $106,000. Although the index increased by 15% and the participation rate of 80% would be a 12% growth rate, the cap of 10% comes into play. That makes the calculation: $106,000 x 110% or $116,600

Which of the following statements under the Investment Company Act of 1940 is TRUE? A) Holding companies are not included in the definition of an investment company. B) Mutual funds furnish financial reports to shareholders at least annually. C) Mutual funds must file semiannual reports with the SEC. D) Investment companies are prohibited from owning more than 5% of another investment company's shares.

The Act lists three different types of investment companies: face amount certificate companies, unit investment trusts and management companies. Holding companies, business entities which invest in other companies for the purpose of management control, are not included in the definition. The limit on investment in another investment company's shares is 3%, not 5%. Section 30(d) of the act requires semiannual reports from the fund to its shareholders and an annual filing with the SEC.

Q. An Administrator has specific authority under the USA to I. suspend the registration of a security if the suspension is in the public interest and the offering has excessive underwriting fees II. issue emergency injunctions to prevent a violation of the act III. enforce subpoenas in the state at request of an Administrator of another state for alleged violations that occurred in another state IV. require that the proceeds from an offering be held in escrow until the issuer receives a certain percentage of the sale of the securities offered

The Administrator may impound the proceeds of an offering in an escrow account until the issuer receives a specified amount. The Administrator may also suspend a security's registration if excessive fees or commissions are charged as part of the offering. State Administrators have the authority to cooperate with each other in enforcing the provisions of USA by ensuring that the subpoenas from other states are enforced. Injunctions are judicial orders that can only be issued by a court of law, not by an administrative agency such as a state securities Administrator

The alternative minimum tax is designed to ensure that certain high-income taxpayers do not avoid all income tax through the use of various tax preference items. Those preference items are added back to the taxpayer's ordinary income on IRS Form 6251 and would include A) interest received from specified private purpose municipal revenue bonds. B) intangible drilling costs in connection with an oil drilling program. C) straight-line depreciation taken on investment real estate. D) long-term capital gains in excess of $3,000 annually.

The Internal Revenue Code provides that interest on specified private activity bonds is an item of tax preference. Therefore, this interest must be added to a taxpayer's regular taxable income in order to compute the taxpayer's AMTI. Accelerated depreciation and excess intangible drilling costs are preference items. In the real world, it is not only those with high incomes that are caught by the AMT, but the exam is not likely to go that deep.

Which of the following is NOT a valuation method for a fixed-income security? A) Conversion parity B) Price-to-earnings ratio C) Dividend discount model D) Discounted cash flow

The P/E ratio is only used with common stock. The parity price is a way to value a convertible bond or convertible preferred stock. DCF is one of the most popular ways to value bonds. The DDM can be used with preferred stock, which, because of its fixed dividend, is considered in the general category of fixed-income security.

A review of an agent's client's account indicates daily trading with rapid portfolio turnover. Under NASAA's Statement of Policy on Dishonest or Unethical Business Practices of Broker-Dealers and Agents, this would NOT be considered excessive trading activity (churning) if A) the client's account shows a profit B) the client has approved each trade C) each security purchased is suitable for the client D) the client's investment objective is quick return, the client has the financial resources necessary for such activity, and the agent uses a sophisticated technical program designed to cut losses and take profits quickly

The Statement of Policy determines whether the trading is excessive by evaluating the client's investment objectives, financial resources, and the character of the client's account. While such trading activity is not suitable for everyone, there are some clients for whom such activity would be suitable. Having client approval for each trade does not necessarily eliminate the requirement to only make suitable recommendations, nor does it excuse excessive trading. A security may be a suitable purchase for a client, but not be suitable for rapid trading. To be sure, there is a greater likelihood of a churning case when the client has lost money, but there have been many cases where even when a profit has been made, churning has been the verdict.

Cecil has a discretionarily-managed account with Pelf Reliable Advisors (PRA), an investment adviser registered in States C, D, and G. Over the past year, the portfolio produced a 12% return with a beta of 1.05. The risk-free rate is 3.5%, and the overall market returned 10.85%. Based on this information, calculate alpha and determine if PRA added any value to the portfolio. A) Alpha = 0.78%; the adviser outperformed the market by 0.78% B) Alpha = -1.21%; the adviser underperformed the market by 1.21% C) Alpha = 0.78%; the adviser underperformed the market by 2.72% D) Alpha = 1.15%; the adviser outperformed the market by 1.15%

The alpha for this portfolio is +0.78% (rounded). A positive alpha indicates that Pelf outperformed the market on a risk-adjusted basis. As with most calculations, there are two ways to solve for the answer. Let's use the LEM's formula first. When the riskfree (RF) rate is given, the formula is (actual return - RF rate) - (beta x [market return - RF rate]). Plussing in the numbers, we have (12% minus 3.5%) minus (1.05 times [10.85% minus 3.5%]). That breaks down to 8.5% minus (1.05 times 7.35%) or 8.5% minus 7.72% = +0.78%. An alternative method is as follows: 12% - [3.5% + 1.05 (10.85% - 3.5%)] = 12% - [3.5% + 7.7175] = 12% - 11.2175 = +0.7825.

An agent is discussing an equity index annuity purchase with a client. The agent explains that there are several that she feels are equally suitable for the client, but one of the companies is offering a trip for 2 to Las Vegas for reaching certain sales goals. She continues by stating that this sale will put her over the goal and win her the trip. If the client purchases that annuity, the agent A) should only sell what is suitable for the client based on all available information B) should pack her bags for the trip; she earned it C) will probably be disciplined for failure to disclose the potential conflict of interest D) should pack her bags and leave the firm before the compliance department learns of her actions

The annuity recommended by the agent is offering an incentive. The agent is clearly disclosing that fact to the client and, if the client goes ahead and makes the purchase, it is with full knowledge of the potential conflict of interest. The question states that the agent considers this annuity, along with others, to be suitable.

Which of the following bonds would most likely be exposed to the greatest amount of interest rate risk? A) JKL 4s of 2020 B) GHI 7s of 2042 C) DEF 6s of 2041 D) ABC 5s of 2040

The bond with the longest duration is generally going to have the greatest exposure to interest rate risk. Because there is very little difference between maturity dates of 2040 through 2042, the bond with the lowest coupon will have the longest duration. The 4s of 2020 have a relatively short duration, even though their coupon is low.

According to the Uniform Securities Act, the investment adviser brochure must include the business backgrounds of A) each member of the investment committee or group that determines general investment advice to be given to clients B) institutional clients C) an affiliated broker-dealer D) all employees of the adviser

The business background of these key individuals must be included in Part 2B of Form ADV and in the disclosure brochure. The business background of other employees, affiliated broker-dealers, and institutional clients need not be included in the brochure.

If you overheard an analyst referring to an investment's indicative value, the discussion would most likely be about A) ETNs. B) ETFs. C) REITs. D) TIPSs.

The calculated value, called the indicative value or closing indicative value for ETNs, is calculated and published at the end of each day by the ETN issuer.

The document that gives the Administrator the right to process complaints against a registrant is known as A) a durable power of attorney B) a consent to service of process C) a writ of habeas corpus D) an injunction Explanation

The consent to service of process gives the Administrator the right to process legal complaints against the applicant.

A feature of which of the following business entities is limited liability but no flow-through of earnings or losses? A) Limited partnership B) Sole proprietorship C) Corporation D) LLC

The corporation (always assume C corporation unless it says different on the test) offers limited liability to its shareholders, but there is no flow-through of income or loss. LLCs and limited partnerships offer both and the sole proprietorship has unlimited liability.

When an investor notices that a bond's coupon yield is lower than its current yield, that is an indication that the bond A) is selling at a discount B) is probably rated investment grade C) is in danger of going into default D) is selling at a premium

The coupon yield, or nominal yield, is the rate stated on the face of the bond. It never changes. However, because the current yield is computed by dividing the coupon rate by the current market price, this return will constantly be in flux. Anytime the price of the bond is below par (selling at a discount), its current yield will be higher than the coupon

The SROs have instituted maintenance margin levels for those situations where the equity in a client's margin accounts is reduced to a dangerous level. Currently, those levels are A) 50% for a long account. B) 25% for a short account. C) 30% for a long account. D) 25% for a long account.

The current minimum maintenance levels set by the SROs is 25% equity in a long margin account and 30% equity in a short margin account. The initial margin requirement under Reg. T is 50% for both long and short accounts.

A customer's limit order to buy 500 shares of QRS at 60 is executed and the agent reports the trade execution to the customer. One hour later, the customer notices that QRS is down 2 points and informs the agent that he no longer wants the stock and is not planning to pay for it. The agent should tell the customer that A) the firm will repurchase the securities from the customer for the price paid B) he may sell the stock at the purchase price in the open market C) he personally will repurchase the securities from the customer for the price paid D) he owns the stock and must submit payment

The customer has entered into a contract to purchase a security as soon as the order is executed. The customer owns the stock and must submit payment.

A customer purchased a 5% U.S. government bond yielding 6%. A year before the bond matures, new U.S. government bonds are being issued at 4%, and the customer sells the 5% bond. The customer probably did which of the following? Bought it at a discount Bought it at a premium Sold it at a discount Sold it at a premium A. I and III B) II and III C) II and IV D) I and IV

The customer purchased the 5% bond when it was yielding 6% (at a discount). The customer sold the bond when other bonds of like kind, quality, and maturity were yielding 4%. The bond is now at a premium. Therefore, the customer realized a capital gain.

A method of assessing the value of a fixed-income security by looking at the future expected free cash flow and discounting it to arrive at a present value is known as A) current yield B) future value C) internal rate of return D) discounted cash flow

The discounted cash flow, DCF, is used to assess the value of a fixed-income security by looking at the future expected free cash flow and discounting it to arrive at a present value. This is basically nothing more than taking the income payments you are scheduled to receive over a given future period and adjusting that for the time value of money.

DERP Corporation's 5% convertible debentures maturing in 2030 are currently selling for 120. The conversion price is $40. One would expect the DERP common stock to be selling A) somewhat below $30 per share B) somewhat above $48 per share C) somewhat above $30 per share D) somewhat below $48 per share

The first step here is to compute the parity price. A conversion price of $40 means the debenture is convertible into 25 shares of the common stock (par of $1,000 divided by $40 = 25 shares). With a current market price of $1,200, the parity price of the stock would be $48. Because convertible securities generally sell at a slight premium over their parity price, the stock should have a current market value a bit less than $48 per share.

Which of the following statements is most accurate regarding the net present value (NPV) and internal rate of return (IRR) on a bond? A) NPV assumes that cash flows can be reinvested at the bond's IRR. B) IRR assumes the cash flows are reinvested at market interest rates. C) NPV assumes the cash flows can be reinvested at market interest rates. D) IRR assumes the cash flows are reinvested annually.

The first step in finding the NPV is to compute the present value (PV). The PV is computed by taking the future cash flows and discounting them by a "discount" rate. That rate is the current market interest rate. So, if NPV is based on PV and PV assumes reinvestment at the discount rate, that assumption must hold true for figuring NPV. In the case of the IRR, that is the yield to maturity of a bond and assumes that the cash flows are reinvested at that IRR. For example, a bond with a YTM of 7% assumes that all reinvestments will be made at that 7% rate. The periodic cash flow on a bond comes from the semiannual interest payments making reinvestments semiannually, not annually.

Assume Frank has a portfolio with an actual return of 10.50% for the past year. The portfolio beta equals 1.25, the return on the market equals 9.75%, and the risk-free rate of return equals 3%. Based on this information, what is the alpha for Frank's portfolio and did it out outperform or underperform the market? A) −1.6875%, underperform B) −0.9375%, underperform C) +9.1875%, outperform D) +3.3750%, outperform

The formula for alpha: alpha = (actual return − risk-free rate) - (beta × [market return − RF])]. If we plug in the numbers, we get (10.5% - 3%) - (1.25 × [9.75% − 3%]) = 7.5% - (1.25 x 6.75) = 7.5% - 8.4375 = -.9375 The alpha for Frank's portfolio equals −0.9375%, indicating that his portfolio underperformed the market based on the level of assumed investment risk.

If the risk-free rate of return is 3.5%, the expected market return is 9.5%, and the beta of a stock is 1.3, what is the required return on the stock according to the capital asset pricing model? A) 7.80% B) 11.30% C) 12.35% D) 8.85%

The formula for the required return is: E(R) = Rf + (E (RM) - Rf) × Beta or 0.035 + (1.3 × [0.095 - 0.035]) = 0.035 + 0.078 = 0.113, or 11.3

An investment adviser representative is evaluating DEF stock to see if it is a good fit for a client's portfolio. Using the security market line (SML), what is the expected return for DEF when the return on the market is 8%, the 91-day Treasury bill is yielding 6%, DEF's beta is 1.50, and the inflation rate, as measured by the CPI, is 4%? A) 8% B) 5% C) 12% D) 9%

The formula for this computation is as follows: 8% (the return on the market is a beta of 1.0) minus the risk-free rate of 6%, or 2%. Then, multiply that by the beta of this stock (1.5) to arrive at 3%. That is, the stock should return 3% over the risk-free rate of 6%, or 9%. Inflation rate is only important if we are looking for the real (inflation-adjusted) return, not the expected return.

The future value of an invested dollar is dependent on 1. the exchange rate of the dollar at the beginning and end of the period 2. the interest rate at maturity 3. the rate of return it earns 4.the time period over which it is invested A) II and III B) II only C) III and IV D) I only

The future value of a dollar reflects the interest rate it earns over time. The rate of foreign exchange is not related to or used in the calculation of the future value of a dollar. The foreign exchange rate is not relevant.

An investment adviser representative is preparing a financial plan for a new client. As part of the data collection process, the IAR needs to collect the relevant information to analyze the client's cash flow. Included in the cash flow statement would be all of the following EXCEPT A) interest on savings B) assets C) income taxes D) salary

The income statement is the basis for an individual's cash flow statement. Rather than assets and liabilities, as would be found on a balance sheet, the concern is measuring income and expenses.

A customer has a nonqualified variable annuity. Once the contract is annuitized, monthly payments to the customer are A) 100% taxable B) 100% tax deferred C) 100% tax free D) partially a tax-free return of capital and partially taxable

The investor has already paid tax on the contributions, but the earnings have grown tax deferred. When the annuitization option is selected, each payment represents both capital and earnings. The money paid in will be returned tax free, but the earnings portion will be taxed as ordinary income.

If 150 investors want to form a corporation to limit their financial liability to the amount of money they invest and do not want to be responsible for any debt that the corporation incurs, they would most likely form A) a proprietorship B) a C corporation C) an S corporation D) a general partnership

The investors would form a C corporation. The advantages of the C corporation are that stockholders are not liable for corporate debt; that it is easier to raise money by issuing stock; that it is easier to transfer ownership; and that unlike a partnership or a proprietorship, a C corporation has a continuous life because it does not terminate on the death of shareholders, officers, or directors. An S corporation is limited to 100 investors.

wo of the major factors involved in the capital asset pricing model (CAPM) are interest rates stock risk premium tax rates market risk premium A) II and III B) II and IV C) I and III D) I and II

The model is made up of two separate components. One component is known as the stock risk premium and is the part of the model reflected by the following formula: (market return − the risk-free return) × beta of the stock. The other component is the market risk premium and is the part of the model reflected by the following formula: (market return − risk-free return). The stock risk premium is the inducement necessary to entice the individual to invest in a particular stock, whereas the market risk premium is the incentive required for the individual to invest in the securities market in general. C A P M (PRENIUM)

A $1,000 bond with a nominal yield of 8% will pay how much interest each year?

The nominal yield (or coupon rate) is the interest rate stated on the bond and is the rate the bondholder promises to pay on the bond until the bond matures. A $1,000 bond with an 8% nominal yield will pay $80 per year in interest.

The present value of a dollar A:indicates how much needs to be invested today at a given interest rate to equal a specific cash value in the future B) is the amount of goods and services the dollar will buy in the future at today's rate price level C) cannot be calculated without knowing the level of inflation D) is equal to its future value if the level of interest rates stays the same

The present value of a dollar will indicate how much needs to be invested today at a given interest rate to equal a cash amount required in the future.

investment advisers who preach the benefits of strategic asset allocation do so because they believe A) over the long run, strategic management will eventually outperform the market B) active management of a portfolio offers tactical benefits C) the market is basically inefficient and there is a strategy that can beat it D) the market is perfectly efficient because stock prices reflect all available information

The primary difference between strategic and tactical asset allocation comes down to the belief by those following the strategic style that it is not possible, over a long period of time, to beat the market. U20LO10

A company currently has earnings of $4 and pays a $0.50 quarterly dividend. If the market price is $40, what is the current yield? A) 1.25% B) 15% C) 10% D) 5%

The quarterly dividend is $0.50, so the annual dividend is $2.00; $2 ÷ $40 (market price) = 5% annual yield (current yield).

GHI currently has earnings of $4 and pays a $0.50 quarterly dividend. If GHI's market price is $40, the current yield is A) 10% B) 5% C) 1.25% D) 15%

The quarterly dividend is $0.50, so the annual dividend is $2; $2 ÷ $40 market price = 5% current yield.

In order to compute an investor's real rate of return on a common stock holding, all of the following are necessary EXCEPT A) appreciation B) inflation rate C) dividends D) marginal tax bracket

The real rate of return is another term for inflation-adjusted return. It is the total return, which is appreciation plus income adjusted for the inflation rate as expressed by the CPI. Tax bracket is necessary to compute after-tax return.

Which of the following statements regarding an agent's registration is most accurate? A) Registration of a broker-dealer in a specific state automatically registers all the firm's agents in that state as well. B) Revocation of the registration of that agent's broker-dealer will result in cancellation of that agent's effective registration. C) If the broker-dealer with which that agent is registered should have its registration revoked, the agent's license will be held by the Administrator and the agent will be required to register with an active broker-dealer within 30 days. D) If the broker-dealer with which that agent is registered should have its registration revoked, the agent may continue to do business only with existing clients and may not acquire any new ones until registered with an active broker-dealer.

The registration of an agent is not effective during any period when he is not associated with a particular broker-dealer registered under the Uniform Securities Act. Therefore, when the broker-dealer's registration is revoked, the agent's license is no longer in effect. The Administrator does not "hold" licenses. Agents must register in each state in which they wish to do business; there is no automatic registration other than for certain officers and partners when the firm first registers.

Section 404(c) of ERISA deals with A) distribution options B) tax qualification of the plan C) eligibility requirements D) fiduciary responsibilities

The requirement for a retirement plan trustee to follow fiduciary standards is found in Section 404 (c) of ERISA.

Risk-adjusted return is calculated by A) dividing the security's return in excess of the risk-free rate by its standard deviation B) dividing the price of the stock by the standard deviation C) dividing the security's price by its beta D) multiplying the return of an investment by its standard deviation

The return from a security can be adjusted for the risk by dividing the security's return in excess of the risk-free rate by its standard deviation. This is the Sharpe ratio.

Which of the following might be used by an analyst to approximate a reasonable price for a common stock? A) The dividend discount model B) Par value C) Book value per share D) Yield to maturity

The simplest model for valuing equity is the dividend discount model—the value of a stock is the present value of expected dividends on it. Yield to maturity only applies to debt securities with a fixed maturity date. The par value of a common stock has nothing to do with its market price. Although fundamental analysts will examine a company's book value per share, it generally has little or no bearing on the current market price of the stock.

Which of the following statements is TRUE? A) The Uniform Securities Act is national law that each state enforces through a state Administrator. B) The state Administrator has authority to amend federal law to meet the needs or special circumstance in his state. C) State Administrators are permitted to establish regulations relating to the registration, testing and fees for broker-dealers, agents, investment advisers and investment adviser representatives. D) The USA only provides for civil liabilities for persons involved in illegal securities transactions in their state, while federal law provides criminal penalties.

The state Administrators are permitted to establish the requirements for broker-dealer, agent, investment adviser and investment adviser representative registrations, including testing, filing, and fee regulations. The Uniform Securities Act is model legislation for state regulation (not federal regulation) that each state may adapt to its own needs. The state Administrator does not have authority to amend federal law to meet special circumstance in his state. Each state can model its legislation on the Uniform Securities Act, but a state cannot change federal law and must abide by federal securities legislation. The USA provides for both civil and criminal penalties for persons involved in illegal securities transactions in their state.

The strong-form efficient market hypothesis (EMH) asserts that stock prices fully reflect which of the following types of information? A) Inside only B) Public, private, and future C) Market D) Public and private

The strong form EMH assumes that stock prices fully reflect all information from public and private (inside) sources. That is why proponents of EMH engage in passive strategies, such as using index funds and ETFs.

The Investment Company Act of 1940 requires certain types of investment companies to compute their net asset value on a regular basis. Excluded from that requirement are A) closed-end management investment companies. B) unit investment trusts. C) open-end management investment companies. D) face-amount certificate companies.

The two investment companies offering redeemable securities, open-end funds, and UITs, must compute their NAV on a daily basis. Closed-end funds can do it daily; many compute every Friday. The concept of NAV makes no sense with a FACC.

In order to be in compliance with the rules, an investment adviser would have to disclose that the firm was acting in a principal capacity when A) the trade is being executed by an officer or partner of the firm B) purchasing shares directly from advisory clients C) directing securities transactions to an affiliated broker-dealer D) engaging in an agency cross transaction

There are 2 principals in every securities trade: the buyer and the seller. In this case, buying shares directly from clients who own those shares places the IA in the position of being one of the principals. This is an action that must be disclosed in writing to the client no later than completion of the transaction. In an agency cross transaction, the firm is acting as an agent—that's the reason for the term.

A foreign private adviser is defined in the Dodd-Frank Act as any investment adviser that has no place of business in the United States. has, in total, fewer than 15 clients and investors in the United States in private funds advised by the adviser. has aggregate AUM attributable to clients in the United States and investors in the United States in private funds advised by the adviser of less than $25 million. holds itself out to the public in the United States as an investment adviser or acts as an investment adviser to an investment company registered under the Investment Company Act of 1940. A) IV only B) I and IV C) I, II, and III D) II and III

There are 4 requirements to be considered a foreign private adviser. Choices I, II, and III are all included, and, if choice IV said - does not hold..., it would have been the fourth requirement. By holding itself out to the public, it can't be a private adviser.

Broker-dealers are required to furnish clients with a fee disclosure document. All of the following are true statements about that document except A) it must be up-to-date. B) changes to the fee schedule may be shown on the firm's website. C) changes to the fee schedule must be announced in advance. D) it must be filed with the Administrator of the state in which the broker-dealer's principal office is located.

There is no requirement that the fee schedule be filed with the Administrator. It must be up-to-date and any changes must be announced in advance (usually a minimum of 30 days). There are a number of ways to disclose the fees, the firm's website is one of them.

An investment adviser wishes to engage the services of a third party to solicit new clients for the firm. To be in compliance with the Investment Advisers Act of 1940, I. the solicitor must be registered as an IAR II. compensation may not be sales related III. the solicitor must not be subject to statutory disqualification IV. disclosure of the solicitation arrangement must be made to clients upon request

Third-party solicitors are not required to be registered as IARs and therefore may not receive sales-related compensation. However, they must not be subject to statutory disqualification that would prevent them from becoming registered. Disclosure is necessary, whether or not it is requested.

In general, the most passive investment style for a portfolio would be A) value. B) contrarian. C) indexing. D) buy and hold.

This is a close call between indexing and buy and hold. We believe that the NASAA philosophy on this would be that buy and hold does require some management after the portfolio is set up. That is, some companies go out of business or are merged into other entities or go private and that requires making new decisions. The same can happen with the companies in an index, but the investor doesn't have to make the changes. When you invest in an index, it is sort of like (with credit to Ron Popeil) "set it and forget it". Clearly, the other two choices are not passive in the same way

ABC Investment Company shares are trading at $13.80 on a per-share basis. The net asset value per share is $12.00. Which of the following conclusions correctly defines the relationship between trading price and NAV? A) The fund's shares are trading at a premium of 15% to the NAV. B) The fund's shares are trading at a discount of 15% to underlying NAV. C) The value of $13.80 is calculated as total assets minus total liabilities divided by total outstanding shares. D) NAV per share is calculated as per-market demand and supply for the fund's shares.

This is a closed-end investment company whose shares are trading at a premium. The premium is 15% relative to the underlying NAV ($1.80 ÷ $12.00). The market price, not the NAV of the fund's shares, is determined by supply and demand in the market. How do we know this is not a mutual fund? There are two ways. Mutual funds do not trade; there is no secondary market for them. Secondly, the sales charge is 13.8% ($1.80 ÷ $13.80) which is far above the maximum 8.5% allowed.

An investor has $100,000 to invest. If the account's estimated annual return is 8% and the investor plans to withdraw $20,000 at the end of each year, approximately how long will the money last? A) 7.55 years B) 6.65 years C) 5 years D) 8.33 years

This is a simple calculation with a financial calculator, but the test center does not let you use one. So, if you get a question like this (and have time - don't waste it if it means you won't get to the easy questions at the end of the exam), you can do it long-hand as shown in the LEM. For those without a LEM, it goes like this: Beginning of year (BOY)End of year (EOY)$100,000× 108%$108,000 − then subtract $20,000$88,000× 108%$95,040$75,040× 108%$81,043.20$61,043.20× 108%$65,926.66$45,926.66× 108%$49,600.79$29,600.70× 108%$31,968.85$11,968.85× 108%$12,926.36 Therefore, the investor can withdraw the planned $20,000 for 6 full years and will have slightly less than enough for 65% of the full payment in the 7th year.

An investor has her agent enter a sell stop order at 60, limit 60. Following the order entry, trades occur at 62.12, 60, 59.95, 60.06, 61. More than likely, the investor received A) 61 B) 60 C) 59.95 D) 60.06

This is really two orders. The first is to "stop" at 60. That is, once the stock trades at 60 or lower, enter my order. That second order is a sell, but with a limit of 60. So the first time the stock hits 60 (or less) is the trade at 60. That triggers the sell limit. The next trade is a 59.95. Because the limit order is saying, "Get me 60 or higher, the 59.95 is not an acceptable price." But, the next trade, 60.06 will meet the client's goal of receiving no less than 60

Under both state and federal law, the executive office of the investment adviser from which the officers, partners, or managers of the investment adviser direct, control, and coordinate the activities of the investment adviser is properly referred to as A) the principal office and place of business B) the home office C) the office of supervisory jurisdiction (OSJ) D) the registered office

This is the way it is defined in the act. It is generally the home office, but you must chose the answer that best meets the terminology referred to in the question. Office of supervisory jurisdiction (OSJ) is a FINRA term and is not applicable on this exam.

One of your clients purchases a European-style put option on a stock. The premium is $3 and the exercise price is $35. If the price of the underlying asset is $40 on the exercise date, the client has A) made $500. B) made $200. C) lost $200. D) lost $300.

This option is out of the money and is therefore worthless. Remember, European-style options are exercisable only at expiration and a $35 put is worth zero unless the market price of the underlying asset is less than $35. As is the case with any long option position, the maximum loss is the premium paid.

The Smiths are saving money for a down payment on a house. The Smiths have $25,000 in cash, and they estimate that in 5 years they will have approximately $31,000 if they deposit their cash in a savings account that compounds interest yearly. To calculate the $31,000 amount, the Smiths determined A) the internal rate of the return on the $25,000 B) the present value of $25,000 C) the future value of the $25,000 D) the net present value of the $25,000

To determine the money's worth at a future date (in this case, 5 years), the Smiths calculated the future value of the funds. Future value is a compounded rate of return, and in this case, the $25,000 was compounded at 5% per year for 5 years. The present value of an investment is the opposite of future value.

According to the Uniform Securities Act, to determine whether an investment adviser is trading excessively in a customer's account, regulators primarily examine whether A) the adviser acted as a principal or an agent B) the adviser received compensation for the trades C) the customer approved the transactions in writing D) the transactions matched the investor's objectives

Trading in a customer's account must not be excessive in terms of size or frequency with respect to the customer's investment objectives and financial ability. For example, if the client's objective was speculative trading, a higher-than-normal volume would be expected. It could also be important to know whether the adviser was receiving trade-based compensation (which could be the reason for the possible churning), but that would be secondary to customer objectives.

A client of a broker-dealer calls his agent and submits an order to purchase 1,000 shares of a Peruvian copper mining company. As the order ticket is being prepared, the agent notices that this is a nonexempt unregistered stock. The agent should A) inform the client that no orders for this stock may be accepted until it is properly registered in the state B) wait for firm approval before processing the order C) continue to process the order because this is an exempt transaction D) continue to process the order because this is an exempt security

Transactions resulting from unsolicited orders are exempt under the USA. Therefore, this order may be taken as placed.

Which of the following are prohibited practices? An investment adviser transferred a client's account to a brokerage house because the account went below the firm's minimum size and then informed the client. An investment adviser organized as a partnership did not inform its clients of the departure of a partner who had only a very small interest in the firm. An investment adviser subsidiary of a publicly traded bank holding company failed to inform its clients of the departure of the firm's chairman and major stockholder. An investment adviser firm organized as a general partnership sends prompt notification to all clients after the addition of a new partner. A) III and IV B) I and II C) II and III D) I and IV

Transfer or assignment of an advisory account without prior client consent is always prohibited. An investment adviser need not inform clients of departures of employees, senior or otherwise, from investment advisory firms that are incorporated. Clients must, however, be informed of the departure or addition of any partner if the firm is organized as a partnership. The legal requirement for this notification is "within a reasonable period of time," but there is nothing prohibited about doing it promptly.

A benefit of active investment in real estate that is not available to purchasers of REITs is A) the Section 1031 exchange privilege. B) the Section 1035 exchange privilege. C) greater liquidity. D) dividends from active investments are generally qualified.

Under Internal Revenue Code Section 1031, no gain or loss is recognized on the exchange of real estate held for investment if such property is exchanged solely for real estate of like-kind which is to be held for investment. This does not apply to REITs where an exchange is considered a sale with a realized gain or loss for tax purposes. Section 1035 is similar in concept, but deals with insurance products, usually annuities. Dividends are paid by corporations, not those who flip houses and, because most REITs are publicly traded, they are the ones with greater liquidity.

To qualify as a REIT, a company must have the bulk of its assets and income connected to real estate investment and must distribute at least what percent of its taxable income to shareholders annually in the form of dividends? A) 50% B) 75% C) 70% D) 90%

Under SEC rules, to qualify as a REIT, a company must have the bulk of its assets and income connected to real estate investment and must distribute at least 90% of its taxable income to shareholders annually in the form of dividends. Don't confuse this with the requirement to have at least 75% of its income from real estate or 75% of its assets invested in real-estate related assets.

Under the Uniform Gifts to Minors Act, Ralph wants to give some stock to his brother's son, Jose. His nephew's father, Bob, is the legal guardian. If Ralph wants to name himself as custodian, which of the following needs to be done? A) Ralph must file the proper legal documents. B) Ralph must have the permission of the guardian. C) Ralph must open the account and name himself as the custodian. D) Ralph must receive legal permission to act as custodian.

Under UTMA or UGMA, no special documentation is required. The account is opened in the name of the minor with the minor's Social Security number and the name of the adult listed as custodian.

If Janet established a Coverdell Education Savings Account for her grandson, in each successive year, she may contribute A) $3,000.00 B) $4,000.00 C) $2,000.00 D) $1,000.00

Under current regulations, the maximum contribution to a Coverdell Education Savings Account is $2,000 annually.

An analyst observes that the beta of a security is 1.3, the market return is 6%, and the risk-free rate is 1%. The analyst forecasts that the security will return 7% over the next year. Based on these assumptions, the security is A) overvalued, because the forecasted return exceeds the required return. B) undervalued, because the required return exceeds the forecasted return. C) undervalued, because the forecasted return exceeds the required return. D) overvalued, because the required return exceeds the forecasted return.

Under the CAPM, the required return is the RF rate plus the beta times (the market return - RF rate). Using the numbers in this question, it is 0.01 + 1.3 (0.06 - 0.01) = 0.01 + 1.3 (0.05) = 0.01 + 0.065 = 0.075 which is 7.5%. Because the forecasted return of 7% is less than the required return of 7.5%, this security is considered to be overvalued. U20LO9

Under IA-1092, an investment adviser 1makes advice his principal activity 2makes advice his regular activity 3is compensated directly for advice 4is compensated directly or indirectly for advice A) II and III B) I and III C) I and IV D) II and IV

Under the SEC's release, the rendering of advice does not have to be a person's principal activity. Rather, it must be a regular activity, and compensation may be received directly or indirectly.

Under the Securities Exchange Act of 1934, an exchange is A) any transaction involving a security B) a disposition of a security for value C) an organization of securities professionals designed to promote fair practices in doing business with the public D) an organization that provides facilities for bringing together buyers and sellers of security

Under the Securities Exchange Act of 1934, exchange does not refer to a transaction but to an organization or facilities for bringing together buyers and sellers of securities. It is important to distinguish this function from other activities carried out by persons in the secondary market, such as transfer agents, securities information processors, or broker-dealers.

Under the USA, a state securities Administrator can 1start an investigation against a registrant even if a violation has not yet occurred 2subpoena witnesses living in the Administrator's state only 3subpoena witnesses living outside the state 4begin an investigation only after a violation of the act has occurred A) I and II B) I and III C) II and IV D) II and III

Under the USA, the Administrator has a broad investigative authority and may begin an investigation against a registrant before a violation has occurred and may subpoena witnesses in any state.

Under the Uniform Securities Act, an offer to sell includes all of the following EXCEPT A) an attempt to offer to dispose of a security for value B) a solicitation of an offer to buy a security for value C) an offer of a warrant to purchase or subscribe to another security D) a distribution of a stock dividend in lieu of a cash dividend

Under the Uniform Securities Act, an offer to sell includes every attempt or offer to dispose of, or solicitation or an offer to buy, a security for value. If a corporation distributes a stock dividend instead of a cash dividend, and the shareholder is not required to make any payment, then issuance of the stock dividend does not constitute an offer to sell.

Which of the following situations would require registration as an investment adviser? 1A broker-dealer provided investment research services to a customer and charged a fee for the service. 2An agent of a broker-dealer recommends the purchase of ABC securities to a customer, who then purchases 100 shares, and the agent earns a commission. 3A broker-dealer has its agents prepare complete financial plans for customers for a nominal fee. The plans recommend specific securities transactions, and when the customers place orders, the agents earn commissions on those securities transactions. 4A broker-dealer charges its customers for collecting dividends and maintaining their accounts in addition to commission charges for transactions executed. A) I only B) I and III C) I, II, III, and IV D) I, III, and IV

Under the Uniform Securities Act, broker-dealers and their agents are not defined as investment advisers if their performance is solely incidental to the conduct of a brokerage business, and no special compensation is received for the advisory services. A broker-dealer charging for research advice is charging for advisory services, which would require registration as an investment adviser. Preparing a complete financial plan for a customer goes beyond being solely incidental to conducting a brokerage business and would require registration as an investment adviser because a fee was charged, even if only a nominal one. Although not asked in this question, those agents would also have to register as IARs. Recommendations of securities purchases are incidental to conducting a brokerage business and would not require registration as an investment adviser if no fees are charged for the advice. Broker-dealers may charge for clerical services provided to customers, but clerical services are not considered investment advisory services.

Agent A with Firm Y and Agent B with Firm Z conduct a joint seminar. They agree to share the commissions on any resulting business. Under the Uniform Securities Act, which of the following statements regarding sharing commissions is CORRECT? A) In this instance, sharing of commissions could only be done with the approval of both firms. B) Sharing of commissions by agents of two unrelated firms is prohibited. C) Sharing commissions that are a result of a joint seminar is never permitted. D) Only an agent who makes a sale is eligible to earn a commission.

Unless an exception is granted by the Administrator, it is prohibited for an agent to share commissions with any person not also registered as an agent for the same or affiliated broker-dealer.

The USA provides either an exclusion from the definition or an exemption from registration as an investment adviser for certain persons. Which of the following would be required to register? A) A bank trust officer with less than $250 million in assets under management B) An engineer employed by an oil company selling limited partnership interests to public investors who provides estimates of recoverable reserves C) A CFP® who provides a full range of financial planning to clients on a fee-only basis D) A teacher who teaches a course in the local high school on consumer economics

Unless excluded or exempted, anyone charging a fee for investment advice must register. Banks and their employees are excluded. Engineers and teachers fall under the late exclusion as long as the advice is incidental to their profession and no special compensation is received.

Active Technicians (AT) is a state-registered investment adviser. In its brochure supplement, it would include information relating to each of the following individuals EXCEPT A) those providing investment advice and having direct contact with retail clients in the state B) members of AT's board of directors who are active in the firm's business C) those exercising discretion over assets of clients in this state, even if no direct contact is involved D) those providing investment advice and having direct contact with institutional clients in the state

Unless the individual has direct contact with clients (retail or institutional) or exercises discretion, a copy of the Part 2B brochure supplement for each individual is not required. This would include officers and members of the board of directors. Of course, if any of these individuals had direct client contact or exercised discretion, a supplement for them would need to be prepared.

Which of the following are characteristics of newly issued warrants? A) Time value and intrinsic value B) Time value, but no intrinsic value C) No intrinsic value and no time value D) Intrinsic value, but no time value

Warrants could be thought of as call options with a long expiration period. They are always issued with a strike price in excess of the current market value, so there is no intrinsic value. One could say that on issuance, they are always out-of-the-money. The only value is in the time to expiration, usually several years or longer.

An increase in the earnings per share growth rate from one reporting period to the next is called A) finding alpha. B) price-to-earnings ratio. C) profitability. D) earnings momentum.

When a company's rate of earnings per share growth is increasing, that is, not only is the company becoming more profitable, but it is doing so at an accelerating rate, analysts call that earnings momentum. The company may or may not be generating positive alpha. We would have to know its beta, the risk-free rate, the market return, and the stock's return to compute that. It is likely that earnings momentum would cause the P/E ratio to increase, but that doesn't answer the question. Earnings momentum is one of the goals of the growth style of investing.

A man divorces his spouse after 10 years of marriage and remarries. If the man is the sole provider, what part of the worker's Social Security benefits is the new spouse entitled to? A) The new spouse is entitled to splitting the benefits with the ex-spouse. B) She will be entitled to the same Social Security benefits as the ex-spouse after 10 years of marriage. C) She is entitled to the same Social Security benefits as the ex-spouse. D) The new spouse is entitled to more benefits than the ex-spouse.

When an individual remarries, the new spouse is entitled to full Social Security benefits. As long as the previous marriage lasted at least 10 years, that ex-spouse (if not remarried) is also entitled to full benefits. That means it is possible for 2 people to receive full benefits at the same time.

Prosperity Asset Partners (PAP) is organized as a general partnership. PAP is registered in four states. All of the following statements regarding the investment adviser brochure rule of the Uniform Securities Act are true except A) the disclosure brochure must be delivered no later than 48 hours before entering into an advisory contract for there to be no requirement to offer a 5-day refund right B) the disclosure brochure must be signed by an officer or a general partner of the firm C) the brochure rule permits advisers to deliver the disclosure brochure when the client enters the contract providing the client is allowed to cancel the contract without penalty within 5 business days D) the disclosure brochure must contain essentially the same information as is contained in Form ADV, Part 2A and, if applicable Part 2B.

When an investment adviser's business structure is a general partner (as is the case with PAP), the brochure must be signed by a general partner. If the firm is a corporation, then an officer's signature is acceptable. The investment adviser's disclosure brochure must contain the relevant information from Form ADV Part 2A and, for those where it applies, Part 2B. The rule does permit advisers to deliver the brochure when the client enters the contract, provided the client is allowed to cancel the contract without penalty within 5 business days; otherwise, the brochure must be delivered no later than 48 hours before entering into an advisory contract.

Thirty years ago, an investor deposited $100,000 into a single premium deferred variable annuity. Today, the value of the accumulation units is $1.5 million. The investor is ready to annuitize and wishes to maximize monthly payments to be received. You would suggest which of the following settlement options? A) Life with 10 years certain B) Life with 20 years certain C) Joint and survivor D) Straight life

When one annuitizes, the amount of the annuity payment is highest when the annuitant takes the most risk (and the insurance company the least). Straight life payments end upon the death of the individual, and if that should be the following month, the insurance company keeps the rest of the money. In the period certain choices, the insurance company is "on the hook" for that number of years, even if the annuitant does not live that long.

The bond strategy used most often by those with a target goal is A) the bullet strategy. B) the laddering strategy. C) the barbell strategy. D) the duration strategy.

When you think of a bullet, you think of it hitting a target. That is what the bullet strategy is all about. When an investor will be making periodic investments in bonds and there is a specific future goal, such as retirement, the bullet strategy seems most appropriate. Some investors might consider a barbell with the target date being in the middle of the short-term and the long-term group, but at least for exam purposes, select the bullet strategy when you are aiming for a target goal.

Which of the following is a factor that must be considered when constructing a portfolio? A) Client's risk tolerance B) Category of investment service required C) Verification of the client's identity D) Performance measurement

While forming a portfolio or investment policy statement (IPS), gauging a client's risk tolerance is the key task.

An Administrator may summarily suspend a registration of an agent or an IAR pending final determination of proceedings under the Uniform Securities Act. However, the Administrator may not enter a final order without 1appropriate prior notice to the applicant as well as the employer or prospective employer of the applicant 2opportunity for a hearing 3findings of fact and conclusions of law 4prior written acknowledgment of the applicant A) I and II B) I only C) I, II, III, and IV D) I, II, and III

With the exception of those proceedings awaiting final determination, the Administrator must provide an appropriate prior notice to the applicant as well as the employer or prospective employer of the applicant and provide the opportunity for a hearing. In addition, the Administrator may only issue a final order after findings of fact and conclusions of law. An applicant is not required to provide written acknowledgment before an order is issued.

corporation calls in a portion of its long-term debt at 101. This will have the effect of I. decreasing working capital II. increasing working capital III. decreasing net worth IV. increasing net worth A) I and IV B) II and IV C) I and III D) II and III

Working capital is computed by subtracting current liabilities from current assets. Using a current asset, like cash, to call in the bonds, reduces those assets with no corresponding reduction to current liabilities. Whenever a bond is called at a premium, net worth is reduced by that premium.

For a bond selling at a discount, the yield to maturity will be A) equal to the nominal yield B) lower than the nominal yield C) higher than the yield to call D) higher than the nominal yield

Yield to maturity is a measure of the total return on a long-term bond, including capital appreciation and interest, while nominal yield measures the interest rate stated on the face of the bond. An investor who buys a $1,000 bond at a discount (for less than $1,000) will receive the interest payments on the bond at the nominal rate and will still receive $1,000 for the bond when it matures. As a result, the total return will be higher than the nominal yield. When a bond is selling at a discount the YTC will always be higher than the YTM.

Expressed as a percentage, what is the total return on a 1-year, newly issued (365 days to maturity) zero coupon debt obligation priced at 95? A) 5.26% B) 5% C) The return cannot be determined without knowing current interest rates. D) 5.26% plus the implied coupon rate

Zero coupon bonds get their name from the fact that there is no coupon interest paid. The investor's return is the difference between the discounted price paid and the $1,000 maturity value. In this question, the price paid was 95 ($950) and the maturity value is $1,000. That $50 difference is the investor's return. To determine the total return percentage on this zero coupon debt obligation, the $50 capital appreciation is divided by the cost of the bond, in this case $950, for a total return of 5.26%. Total return of a zero coupon security is made up entirely of the difference between the cost of the security and its sale or maturity price. The market price of the security, not current interest rates, is used in the calculation of total return.


संबंधित स्टडी सेट्स

The Julio-Claudians/ Civil War and The Flavians/ The 5 good emperors

View Set

Adult Med- Surge GI: Constipation - Surgeries

View Set

Health and Wellness 110 Final Review

View Set

Identifying: X Intercepts & Y Intercepts

View Set

Introduction to Networks chapter 1-16

View Set